ОГЭ
консультация (9 класс)

Бадмаева Татьяна Владимировна

Подготовка к ОГЭ по английскому языку, 9 класс

Скачать:

ВложениеРазмер
Файл grammatika_i_leksika.docx52.88 КБ
Файл voprosy.docx21.19 КБ
Файл pismo.docx21.59 КБ
Файл chtenie.docx96.33 КБ
Файл 3_zadanie_oge.pptx541.92 КБ
Файл teksty_dlya_chteniya.docx30.6 КБ
Файл lichnoe_pismo.docx14.73 КБ

Предварительный просмотр:

Преобразуйте слова, напечатанные заглавными буквами в конце строк так, чтобы они грамматически и лексически соответствовали содержанию текста. Заполните пропуски полученными словами. Каждый пропуск соответствует отдельному заданию.

  1. What is the largest animal on the planet? My uncle says that it’s the blue whale. He is a ______SCIENCE______ and knows a lot about them. The blue whale is an endangered animal, nowadays, people protect whales.   ______FORTUNATE______, They study these______BEAUTY______ animals instead of killing them. Blue whales are not ______DANGER______ to people. It’s______BELIEVABLE______ but these huge whales eat plankton – small plants and animals.

  1. It was getting dark when Laura got to her sister’s place. The entrance door ______LOCK______.

Laura pressed the doorbell and ______HEAR______ quick footsteps. “It’s so nice of you to come,’ Laura’s sister Betty opened the door.

“Come in. We ______PUT______ a meat pie into the oven. I think it’ll be ready in forty minutes or so. You aren’t very hungry, are you?”

Betty was seven years ______YOUNG______ than Laura, but she was much better at housekeeping.

When Laura entered the house, she saw four ______WOMAN______ in the sitting room.

They nodded and went on talking. “Where’s Ken?” asked Laura. “He ______REPAIR______ his car in the garage,” Laura said. “He ______JOIN______ us soon.”

  1. Dolphins are ______WONDER______ animals.

They are clever and ______FRIEND______, and they don’t mind playing with people.

My father is a ______SCIENCE______ and he has heard how dolphins speak their own language.

Dolphin ______COMMUNICATE______ differs from ours, but scientists hope that soon we’ll be able to understand each other. The dolphins are ______PROBABLE______ studying us too!

  1. The weather was unusually hot. Larry was sitting in the shade of the only tree in the garden, with a magazine in his hands. He was trying to read but ______NOT/CAN______ concentrate on the text.

All the ______CHILD______ had left the city, but Larry had to stay at home.

He was daydreaming when he ______HEAR______ Jimmy’s voice:

“It’s ______HOT______ today than it was yesterday!

If it goes on like this, everyone ______BELIEVE______ in global warming.” Larry looked up. Jimmy was holding a paper propeller in his hands.

“Larry,” Jimmy sounded excited, “Look what I ______MAKE______! It’s a model of a super machine to save people from heat._________ you ______LISTEN______ to me? It’s new technology!”

  1. Life on the planet would be ______POSSIBLE______ without water.

Whatever your ______FAVOUR______ drink is, you can’t make it without water. We need water for cooking, washing and producing goods.

My uncle, who is a ______FARM______, says that he waters the vegetables, beans and other crops a lot in dry weather.

______FORTUNATELY______, fresh water resources on the Earth are limited.We should be ______CARE______ and not waste it.

  1. People of all ages like cartoons. We went to the cinema yesterday to see an action film when, suddenly, I ______SEE______ a poster for a cartoon.

I’m the ______YOUNG______ among my friends, so I wasn’t sure they’d want to see the cartoon too, but they did. Even Mike didn’t mind.

“I ______SEE______ the action film anyway. Let’s watch a cartoon for a change,” he said.

It was a story about four ______MOUSE______.

They saved an injured cat that ______CALL______ Bart.

The cat recovered but ______NOT/WANT______ to leave his new friends.

They had funny adventures together. “I think I ______WATCH______ the cartoon again, together with my girlfriend.” Mike said on the way home.

  1. I love music. Since my ______CHILD______,

I’ve played many ______DIFFER______ musical instruments: the piano, the guitar, and even the drums.

My music ______TEACH______ sometimes says that I have a gift for music, and I hope she’s right.

______RECENT______, I’ve joined our school band and we are going to take part in a modern music concert.

______FORTUNATELY______, we couldn't participate in the city music competition as it was our exam time.

  1. Kate is a serious girl. Seven years ago she won first prize in the school singing competition. However, the career of a singer ______NOT /ATTRACT______ her.

She found science ______INTERESTING______ than singing.

Kate ______THINK______ that she wanted to study physics.

Her parents were surprised but did everything they ______CAN______ to help her.

The family moved to the city, where Kate joined the Physics Experimental Laboratory for gifted ______CHILD______.

Now, she is making her further education plans, and her parents think she ______HAVE______ a brilliant career in science. “I know what I want to do. I ______CHOOSE______ the university I’m going to enter,” Kate said to the journalists.

  1. My mum enjoys gardening and I like helping her. Mum says that my grandfather was a ______FARM______ and that he loved working with the earth. ______FORTUNATELY______, I didn’t know him as he died before I was born.

We grow ______BEAUTY______ flowers and fruit trees.

We have some garden furniture – a ______WOOD______ table

and four ______COMFORT______ chairs. The neighbours often come round, and we have outdoor tea parties.

  1. My mum enjoys gardening and I like helping her. Mum says that my grandfather was a ______FARM______ and that he loved working with the earth. ______FORTUNATELY______, I didn’t know him as he died before I was born.

We grow ______BEAUTY______ flowers and fruit trees.

We have some garden furniture – a ______WOOD______ table

and four ______COMFORT______ chairs. The neighbours often come round, and we have outdoor tea parties.

  1. My parents want me to go to university but I’m not sure that I want to. When I was a small child, there were lots of ______DIFFER______ toys in our house, but I preferred the cars.

I dreamed of being a ______DRIVE______.

To me, it’s the most ______INTEREST______ job in the world.

My parents ______USUAL______ understand me

and I hope they approve of my ______DECIDE______ to put off university and get a job.

  1. It was Sunday so I didn’t have to go to school. I woke up late, got up and ______GO______ to the kitchen.

It was strange but I ______NOT/CAN______ find our cat, Tom, anywhere. The whole family got worried.

“He’s gone hunting,” my ______YOUNG______ sister said.

“All cats hunt ______MOUSE______. I read about it in a book.”

“I’m sure he ______SLEEP______ in the armchair,” Mum said. But the cat wasn’t there either.

“Look! I ______FIND______ him!” my little sister shouted. “He’s in the washing machine!” We watched in surprise as the cat got out of the washing machine.

  1. “Come on, kitty, come here. We ______GIVE______ you some milk.”

Mum wants to buy a new sofa for the sitting room. So, yesterday, we went to the largest furniture store in the city. However, we didn’t buy anything because Mum couldn't make a ______DECIDE______ about what she liked best.

There were sofas in ______TRADITION______ and modern styles, in ______DIFFER______  colours and patterns.

The shop ______ASSIST______ spoke very patiently and showed us one model after another.

At last Mum chose one of them, but, ______FORTUNATELY______, they didn’t have that model in her favourite colour.

  1. Lisa was walking slowly along the aisles. The University library was the ______LARGE______ library she had ever seen.

The librarians moved quietly as they were wearing special soft shoes on their ______FOOT______.

 “Can I help you?” A young librarian ______COME______ up to Lisa.

“Yes, thank you. I ______FIND______ all the books from my reading list,” Lisa said, “but I can’t find any information about architecture.”

“I ______SHOW______ you,” the librarian said.

“By the way, ________ you ______KNOW______  who built our library?” Lisa didn’t.

“Oh, it ______BUILD______ more that four centuries ago. I can recommend a book about the architect if you are interested.”

  1. When I was a small child, we often went to the circus. I loved the ______FUN______ clowns and the motorbike ride in the globe.

The boy who did the riding seemed only a bit older than me. He performed the trick very ______CAREFUL______.

I certainly thought that it was extremely ______DANGER______, anyway. I applauded the ______BIKE______ loudly and dreamt about riding my own motorbike.

______FORTUNATELY______, my parents had a different opinion about getting a motorbike and refused to buy one for me.

  1. Sophie’s flight was delayed. She was waiting in the departure lounge feeling bored. She tried to read but ______NOT/CAN______ concentrate on the text.

Men and ______WOMAN______ with bags and suitcases crowded near the information desk and near the restaurant.

Sophie ______NOT/WANT______ to eat.

She pulled out her mobile and called her mother. “It’s me, Mum.  I ______SIT______ in the airport. No, it’s not cold here.

Yes, I ______CHECK______ in.” Sophie’s mother had four children.

Sophie was the ______YOUNG______ of them – her mother worried about her the most. Suddenly, Sophie saw a familiar face in the crowd of passengers.

“I’m sorry, Mum, I have to go now. I ______CALL______ you later. Don’t worry!”

  1. We went to France last weekend to visit my grandma. She lives in the countryside in a nice ______TRADITION______ house.

I like her house very much. There’s old ______WOOD______ furniture in the rooms and ______WONDER______ pictures on the walls.

Grandma has a large ______COLLECT______ of old pictures. Some of them were painted by an artist who lived next door to her.

He was very talented but, ______FORTUNATELY______, he died young and didn’t become famous.

  1. Andrew hated shopping. However, he had enjoyed it on one occasion. It happened when his two aunts came to his city and Andrew ______HAVE______ to entertain them.

Andrew thought that most ______WOMAN______ like shopping.

So, he took his aunts to the ______LARGE______ shopping centre in the city.

However, they ______NOT/BUY______ anything.

“It’s a nice place,” aunt Laura said, “but I want to have a break from hunting for clothes. __________________ you ______WANT______ some ice cream, Andrew?”

“And I ______HAVE______ a coffee, if you don’t mind,” aunt Lucy added.

They found a cafe where really delicious ice cream ______SELL______ The aunts told Andrew anecdotes from their school days that made him laugh a lot.

  1. I like reading and watching films about animals and wildlife. I saw a TV series about polar bears recentlyThese ______WONDER______animals live in the northern Arctic. It’s ______BELIEVABLE______, but they feel very ______COMFORT______ in freezing cold weather. A polar bear is a very good ______HUNT______ – it dives into the cold ocean water to hunt seals.

______FORTUNATELY______, global warming is very dangerous for polar bears. They may die out if the temperature rises too high.

  1. When Allan got to the airport, the airport official said: “I’m sorry, sir, but your plane ______TAKE______ off.

You will have to take the next flight. You ______ARRIVE______ in New York at about midnight.”

Allan was very upset: “I must be in New York at 6pm. I have a job interview there. I can’t miss it. It’s the ______GOOD______ job in the world.

Thousands of men and ______WOMAN______ dream about such a job.”

The airport lady ______SAY______ nothing and turned to another passenger.

Allan ______NOT/KNOW______ what to do. He pulled out his notebook.

The time of the interview ______WRITE______ there: 6pm, Tuesday. Allan looked at his digital watch and read: 2pm, Monday.

  1. I do sports ______REGULAR______ Life is impossible without motion and people can’t live if they are not active. I’ve been into sports since my childhood.

When I was seven, a karate ______TEACH______, who trained my elder brother, said that I should

exercise a lot to look sporty and ______ATHLET______.

He was right – I was too fat and looked HEALTHY.

______FORTUNATE______, my parents and I followed his advice and now I’m quite happy with the way I look and feel.

  1. Kate knocked on the door and entered the room. The room was large – three times ______LARGE______ than Kate’s classroom.

The walls ______PAINT______ light blue.

Several ______CHILD______ were sitting in a circle.

The teacher was standing beside them. “Come in, Kate,” the teacher ______TELL______ her.

“We ______PLAY______ word games. Would you like to join us?”

Kate ______NOT/KNOW______ how to play word games but sat down between a tall blond girl and a dark-haired boy.

“I ______NEVER/PLAY______ this game,” Kate whispered to the girl. “Is it difficult?”

  1. John was about to leave the classroom when he saw a book on the floor. He opened it and ______SEE______ pictures of mountains and rocks.

He ______NOT/KNOW______ whose book it was. At that moment, Ken rushed into the classroom:

“I ______LOSE______ my book somewhere here!” “Here it is,” John said, “but what’s it about?”

“It’s the ______INTERESTING______ book about mountains I’ve ever seen”, Ken was happy to get his book back.

“It ______WRITE______ by a mountain climber a year ago.

He wrote about how he and three other ______MAN______ went to the Alps to climb.

You can borrow this book for a while if you want. I ______READ______ it anyway.”

  1. Most people prefer travelling by air, but I like trains. A train is slower, but it’s more ______COMFORT______.

My parents think so too. Our longest journey lasted for three days. It was ______EXCITE______ as we went past beautiful places.

The other passenger in our compartment was a geologist, which means he was also a professional ______TRAVEL______.

He told us about his expeditions. He also said that ______FRIEND______ is the most valuable thing in the world.

______FORTUNATELY______, we didn’t exchange phone numbers and we don’t know where he’s now.

  1. Linda read the email and laughed happily. “It’s the ______GOOD______ news I’ve ever got,” she said to her mother.

“I ______PASS______ the exam! It was really difficult, but I managed it!

“Congratulations!” her mother smiled too. “We should celebrate. I ______MAKE______ a huge apple pie. Do you want to invite some friends?”

Linda ______NOT/KNOW______ what to say. “No,” she said at last. “I want to spend this evening with you and my sisters.”

She looked at an old photo on the wall. It ______TAKE______ at the seaside ten years ago.

Linda and her sisters were small ______CHILD______ there.

“I’m lucky to have a family like ours,” Linda ______THINK______ to herself.

  1. People don’t read much nowadays, but I like reading. I like books about adventures, love and ______FRIEND______.

I can’t say who my favourite ______WRITE______ is – there are so many.

Most of my friends prefer watching the TV version of a book to reading it. They say it’s more ______INTEREST______.

I strongly ______AGREE______ with them.

I’m absolutely sure that books develop our ______IMAGINE______ much better than any film can do.

  1. Judy was afraid of the dark. At night she always _________LEAVE_________ the lights on. Judy couldn’t explain what exactly she was afraid of. She ________NOT/BELIEVE__________ in monsters or in ghosts.  She was one of those rare girls who never screamed when she saw spiders, _________MOUSE_________ and frogs. But everyone in her family knew that a dark room was the _________BAD_________ of her fears.  One day, when Judy _________DRAW_________ something in her room, her mother came in holding a big shopping bag. 
    “Guess what I _______BUY______,” she said to Judy.
    “I have no idea. Is it for ________I________?” Judy felt very curious about the bag.
    Her mother pulled two packages from her shopping bag. In the first package there were some very nice slippers. They _________MAKE_________ from soft, pink fabric and had bright flashing lights. Inside the second package were glow-in-the-dark pyjamas! Judy said she _________WANT_________ to try them on immediately. Amazingly, she felt safe all through the night. In a few days the lights stopped flashing but Judy realised she didn’t feel afraid of the dark any more. 

  1. It all started in 1865. A group of Frenchmen were having dinner in one of the most _________FASHION_________ restaurants near Paris. They talked about the democratic changes in the USA and wanted to support the American people. After a short _________DISCUSS_________ they agreed on an original idea. The idea was suggested by Frédéric-Auguste Bartholdi, a sculptor and ________DESIGN_________.  He said they could make a statue of a woman holding the torch of freedom. It was an ambitious project and it was _________POSSIBLE_________ to complete it quickly. It actually took 21 years for this idea to become a reality. The French people raised money to build the statue, and, _________FINAL_________, in 1886 it was ready. On July 4, 1886 the United States received their _________AMAZE_________ gift - the Statue of Liberty! 

  1. As a child, Steven lived in a village. Outside the village there was a house where a strange old woman and her granddaughter Eliza lived. The ________CHILD__________ often played together. One day, Eliza _________BE_________ unusually sad. When Steven was about to leave, the old woman said, “I know you are friends. But never come to our house again. Never ever!”  
    The girl took the boy to the gate. “Will you miss me?” she asked. The boy said, “Yes, I will!” To his surprise, the girl said, “But I won’t! I _________NOT/WANT_________ to see you anymore!” She closed the gate. The next day the girl ________LEAVE__________ for the city. Steven was upset but he hoped that one day he _________SEE_________  Eliza again. Five years later Steven became a student and moved to the city. He rented a small apartment on the _________ONE_________ floor. One day, when he _________WALK_________ to the university, he saw Eliza. She was sitting on the bench in the park with an ice-cream in her hand. He ran up to her and said “Hello! It’s nice to see you again!” But Eliza looked up at ________HE_________, stood up and walked away.  
    “I wish I _________KNOW_________ what is going on,” Steven said to himself, and decided to start his own investigation. 

  1. Choosing a career is one of the most difficult choices we have to make in our lives. There are lots of interesting and _________EXCITE_________ professions, but which of them is the right choice for you? Researchers who work in _________EDUCATE_________ and sociology asked people across the UK what their dream job would be. And the results were quite _________EXPECTED_________. According to the survey, people find modest jobs like charity work or gardening the most _________ATTRACT_________. Often at the top of their list are jobs such as a nurse or a _________TEACH_________. The study also shows that people in the UK are very _________PRACTICE_________ and focus on dream jobs they have a realistic chance of getting. 

  1. A few years ago I was on a camping holiday in Bristol. There _________BE_________ lots of other girls from around the UK, but I was the only one from Scotland. The camp _________LOCATE_________ in the woods, not far from the river. The camp life was even _________INTERESTING_________ than I had expected. We enjoyed all kinds of camping activities –  sports and games, different shows and contests. Everything was fine apart from one incident that took me aback.
    Once, at lunchtime, when I ________EAT__________ ice cream, one of the girls asked: 
    “So, do you have ice cream in Scotland?”
    I _________NOT/KNOW_________ how to react. My neighbour repeated her question. 
    I nodded, “Yes.”
    My answer provoked a wave of absurd questions about my country: 
    “Have you ever seen a ghost in your castle?”
    “Do all Scottish _________MAN_________ wear checked skirts?”
    I __________FIND________ out that some people believe silly stereotypes about Scotland. They believe that Scots really carry bagpipes in ________THEY__________ bags to play the instrument on the streets, and that Scots practise magic. At last, the girls said that they _________COME_________ to Scotland one day to see it with their own eyes and to separate the truth from the myths.
  2. Yoga is a very popular activity and lots of people practise it nowadays. There is an opinion, however, that to be _________SUCCESS_________ in this activity you should start yoga in early childhood. This is not true! You can start yoga at any time in your life. What you really need is enthusiasm and a good ________TRAIN__________ – yogis often call them a guru. Yoga is not a _________COMPETE_________. You can choose any pace that is ________COMFORT__________  for you. The aim is not to win but to make your body flexible and strong. It’s definitely _________POSSIBLE_________ to improve your body in a few days. It may take lots of time but it’s worth the time and effort. Yoga is an _________EFFECT_________ way to relax your mind.  It is useful not only for your body, but for your mind, too. 

  1. July 17th was coming. It would be Mark’s _________THIRTEEN_________ birthday.  His Mum asked him what he _________WANT_________ as a gift. Mark answered at once that his dream was to have a telescope to watch the stars. Mum _________SHAKE_________ her head doubtfully. She knew that Mark enjoyed learning about space but a telescope was a _________EXPENSIVE_________ present than they had planned. After dinner the parents had a discussion about the present. 
    “I think we should give Mark the telescope,” Dad said. 
    “But we ________SPEND__________ all our savings already,” Mum responded.
    “We bought some new furniture and paid for our holidays. I wish we _________CAN_________ buy him this present but we really can’t.”
    “I think, we can,” Dad smiled. “Look, I’ve got an old friend, Nick, whose father was interested in astronomy. His father died many years ago, and Nick keeps all his things in the attic. I think he _________AGREE_________ to sell us his telescope at half price.”
    Nick ________PLANT__________ roses in the garden when Mum and Dad arrived at his house.
    “Yes, sure,” he said. “There are lots of _________BOX_________ in the attic. You can take whatever you want for Mark for free. I’m glad he’s taking up my father’s  hobby.” 

  1. At school, writing essays was a real problem for me, and my grades were always low.  _________NATURAL_________ enough, I didn’t like writing and tried to avoid writing lessons at all costs. I thought I didn’t have enough _________IMAGINE_________ for any kind of writing. I only understood what my problem was after school. In a _________SCIENCE_________ journal I saw an article about children’s writing problems. The author claimed that anyone could become a good ________WRITE__________ if they learned some rules.  I read and reread the article several times. Then I tried to use the information in practice. I have never become a professional journalists but from time to time I publish some pieces of my _________CREATE_________ writing on my blog. My friends find these essays and stories very ________ENJOY__________. 

  1. Olivia got a camera for her birthday. Her family was going on a trip to Washington D.C. and Olivia wanted to take pictures of the _________WONDER_________ places she would see. The first place where they stopped was a sandy beach. Olivia took pictures of the ocean and the _________NOISE_________ seagulls. The birds seemed to enjoy being photographed, but it was _________POSSIBLE_________ to go too close to them. Olivia had to take pictures from a distance. Next, the family moved on to the centre of Washington D.C. Olivia's heart was beating with ________EXCITE__________. She got great shots of the White House, the Capitol and some other _________FAME_________ buildings. _________FINAL_________, it was time to head home. Olivia thought about how her friends would be surprised to see her pictures.

  1. The 5th of November is Guy Fawkes Night. This holiday is also known as Bonfire Night as lighting bonfires is an essential part of the ________CELEBRATE__________. _________NATURAL_________, it’s one of the favourite holidays for children –  they simply can’t miss the firework display. Most parents allow their children to stay outdoors till late at night. It would be _________FAIR_________  to make them stay at home when the sky is lit up with colours. However, children should never be left on their own - bonfires and fireworks are potentially ________DANGER__________. Emergency reports say the number of people needing medical care increases on this night. Some of them have been _________CARE_________ with fire. In spite of the excitement, the children may also catch a cold on this November night. Warm jumpers, coats and gloves are more ________USE__________  than ever. 

  1. Last year, I became a volunteer in a charity shop. It was an _________USUAL_________ job for me - I had never been involved with charities. Our shop raised money to provide medical care for anyone suffering from any ________ILL__________. The job was not easy but I enjoyed it very much. All my colleagues were very helpful. Their _________FRIEND_________ and support meant a lot to me. Andrew, my boss and _________MANAGE_________, was a smart and well-organized person. He kept saying that we had no right to be indifferent or ________CARE__________ because people’s lives depended on us. Working in the charity shop, I have met some very interesting people and have learnt a lot of practical skills. Volunteering has equipped me with experience which I’m sure will be _________USE_________ for my future. 

  1. It was a hot summer day. Tom and his Mum were heading to the beach. Tom was only wearing shorts and a pair of sandals on his ________FOOT__________ but he felt hot anyway. When they came to the beach, they found out that the sea _________NOT/BE_________ calm –  powerful waves were breaking on the shore. Mum _________STOP_________ in hesitation.
    “It’s fine. I can easily swim in these waves,” Tom wanted to demonstrate his swimming skills. But Mum said that they ________STAY__________ out of the water till the sea calmed down.
    Sitting close to ________THEY__________ was a boy about Tom’s age. He was alone because his parents _________LEAVE_________ to buy some sandwiches. He came over and said his name was Gary. His shoulders _________COVER_________ with protective suntan lotion, but they looked red anyway.
    “It’s getting __________HOT________ every minute,” said Gary. “How about a swim?”  
    “No way,” Tom’s mother was definitely against the idea and Tom had to obey.
    “As you like,” Gary said and headed to the water. 
    The ________THREE__________ wave hit him hard and he lost his balance. Everything was happening very fast – Gary fell down and went under the water. Fortunately, Tom and his mother were close enough to get him out in time. 

  1. Last year I decided to take an English course in Britain. When I called the language school, the _________MANAGE_________ explained to me everything about the school and the courses. They had an _________IMPRESS_________ range of courses - for all levels and ages. As for the accommodation, there were two options: staying in a student hostel or living with a host family. Naturally enough, I wanted to choose the _________NATIONAL_________  community of the hostel. However, because of my parents’ strong _________AGREEMENT_________ with my choice, I had to stay with the host family. And I didn’t regret it! My host family was ________WONDER__________. They told me a lot about their country and were very curious about mine. We talked a lot and when I came back home, I could speak English ______FLUENT______! 

  1. Cindy liked parks. She took long walks in the nearby park almost every day. Her classmates, however, _________NOT/SHARE_________ her interest and Cindy usually walked alone. Cindy’s parents worried that she _________HAVE_________ no friends. 
    “I wish you _________SPEND_________ more time with your classmates. Don’t you feel bored in the park?” 
    “It’s ok, Mum, I just feel much ________GOOD__________ alone than with people who can’t understand me,” Cindy usually answered. “Parks are a lot more fun to spend time in than hanging out in a shopping centre.” And her mother didn’t  know what to say. One day, Cindy returned from the park very excited.  
    “Guess, who I met in the park!” she said, and she told her parents the whole story. “I _________WALK_________ along the central avenue when I saw a group of people. They ________BE_________ students of the Biology faculty of the university. They told _________I_________ about the plan to rearrange the park.  A large lake is going to be made in the centre. There will be wild animals there like hares, squirrels and even _________FOX_________. Yesterday, the plan _________APPROVE_________ by the City Council and work starts tomorrow.  I want to join the volunteer team!” 

  1. The academic term was coming to its end and Kate decided to arrange a party to celebrate the coming holidays. To be frank, there _________BE_________ one more reason for the party. Kate wanted her classmates to meet someone from her family.  She was sure it would impress ________THEY__________ and would help her popularity. Kate _________MAKE_________ sandwiches in the kitchen when Hugo came in.
    “It looks like I’m the _________ONE_________ guest, doesn’t it?” Hugo looked embarrassed. “Sorry for coming so early.”
    “Oh, it’s ok,” said Kate, “Come in! You ________HELP__________ me in the kitchen, won’t you?”
    “Sure,” Hugo entered the room and _________STOP_________ in surprise. “Are you saying you made all this yourself?”
    On the table there was a huge pie. It ________DECORATE__________ with different fruit and looked fantastic.
    “No,” Kate looked pleased with his reaction. “My cousin helped me. She ______LEAVE_______ for the shop. She’ll be back any minute.”
    The front door opened and a girl entered. Hugo understood that he ________SEE_________ her many times in different TV programmes for teenagers.
    “A TV star?” Hugo sounded so silly that both girls burst out laughing. 

  1. I arrived in Liverpool in the afternoon. It was a _________RAIN_________ day but the weather couldn’t spoil my excellent mood – I was in Liverpool! Many people associate this city with the _________FAME_________ musical band, the Beatles.  
    It’s true, the Beatles are everywhere, but today it’s also _________POSSIBLE_________ to imagine Liverpool without its other great passion: football! The city is home to two teams: Liverpool and Everton.  Everton is older and was more _________SUCCESS_________ in the past, but since the 1990s it has been Liverpool’s time to shine. I’ve been Liverpool’s _________SUPPORT_________ since I was a small boy. The team is based at the Anfield stadium. I have watched games there twice.  It was very _________EXCITE_________ and it was one of the most important memories of my childhood. 

  1. Tim liked going to the zoo. One day his mother said:
    “We’re going to a different place today, Tim. It’s an animal shelter. I’m sure you _________LIKE_________ it.”
    The place was much _________SMALL_________ than a zoo, but clean and well kept. There were volunteers in special uniforms. One of them showed Tim and his mum round the shelter. Lots of animals ________KEEP__________ there. Cats and dogs made up most of the population, but there were also horses, monkeys, _________FOX_________ and other animals. Each of the animals had its own story written on a sign next to its cage or enclosure. All the stories were sad –  the animals had either been abandoned by _________THEY_________ owners or saved by the animal police from cruel treatment. 
    ‘If people ________BE_________ more responsible for their pets, we wouldn’t have so much work here,” the volunteer remarked.
    “But unfortunately some people ________NOT/UNDERSTAND__________ that animals are not toys.” 
    Tim’s mother thanked the volunteer and _________MAKE_________ a donation. Shelters always needed money to pay for the animals’ food and medical care. On the way home Tim told his mother that he ________WANT__________ to go to the shelter again, not only to watch but also to help the volunteers. 

  1. Last year I learnt Spanish in Barcelona. Spain is an _________AMAZE_________ country and I enjoyed my time there very much. My flatmates were two girls from Germany. They were nice but _________COMMUNICATE_________ was a real problem for us because I didn’t speak German and they didn’t speak English, my first language. Sometimes we felt absolutely ________HELP__________ because we were not able to understand each other. However, it gave us additional motivation to learn Spanish! Very soon my teacher told me that I was speaking it _________FLUENT_________. He was happy with my progress. My flatmates’ grades were _________IMPRESS_________ too! It proved that practice is very important and if you want to speak a foreign language like a native _________SPEAK_________, you should practise it a lot.

  1. Sam went to the giant aquarium near his house at least three times a week. He liked the fish and crabs, but the most _________WONDER_________ creatures there were sharks. Sam could watch them for hours. He took photos through the glass and collected _________INFORM_________ about them. It surprised him how fast a massive shark could be. Before the aquarium opened, Sam thought it was ________POSSIBLE__________ for sharks to be so quick and graceful. He knew that sharks were _________DANGER_________ but they seldom attacked people. Many people, however, had hunted them and sharks had become endangered. _________FORTUNATE_________, now there were laws protecting sharks. In the future, Sam wanted to be a _________SCIENCE_________  and study sharks in the wild.

  1. It was the middle of November and Tina felt down and depressed. Late autumn was the _________BAD_________ season of the year. It often _________RAIN_________, and Tina had to spend most of her time indoors. Because of this, she really welcomed the changes in her school routine. The school administration introduced a new class - a dance class. But not everyone was happy! The boys said that it ________BE__________ a waste of time. They often missed the class and, when they attended, they _________NOT/LISTEN_________ to the teacher’s instructions but tried to make fun of everything. 
    “I wish we _________HAVE_________ a class for girls only,” Tina thought. 
    The Junior Dance Competition _________HOLD_________ in their town every year. It was a spectacular show.
    “Your dancing _________IMPROVE_________ every day,” the teacher said one day.
    “If you go on like this, you _________HAVE_________ a chance to take part in the Dance Competition next month.”
    Tina imagined herself wearing a beautiful dress and elegant shoes on her _________FOOT_________. She decided it was something she’d like to try. 

  1. Olivia got a camera for her birthday. Her family was going on a trip to Washington D.C. and Olivia wanted to take pictures of the _________WONDER_________ places she would see. The first place where they stopped was a sandy beach. Olivia took pictures of the ocean and the _________NOISE_________ seagulls. The birds seemed to enjoy being photographed, but it was _________POSSIBLE_________ to go too close to them. Olivia had to take pictures from a distance. Next, the family moved on to the centre of Washington D.C. Olivia's heart was beating with ________EXCITE__________. She got great shots of the White House, the Capitol and some other _________FAME_________ buildings. _________FINAL_________, it was time to head home. Olivia thought about how her friends would be surprised to see her pictures.

  1. There is a popular opinion that all children are talented. It’s true - children are curious and have a good _________IMAGINE_________. It helps them make _________AMAZE_________ things from very routine objects like stones, leaves, or strips of fabric. Coloured paper, watercolours and pencils stimulate their _________CREATE_________ minds a lot too! However, sometimes parents may _________APPROVE_________ of their children’s artistic activities like drawing on the walls and indoor experiments with fireworks. But _________PUNISH_________ is not a good solution! Any art activity develops children’s brains and stimulates their ability in science. Be _________CARE_________ with your children’s natural talents. Never stop them doing an activity but instead create an appropriate and safe environment for it. 

  1. Annie was waiting for her guests. Her birthday was on 25th December and she felt this was both lucky and unlucky. It _________BE_________ great to have her birthday on a special day like Christmas. But, on the other hand, very few of Annie’s friends came to her party. Most of them had family plans for that day.  The doorbell _________RING_________ and her classmate, Brian, appeared. He had a plastic bag in his hand. 
    “Happy birthday, Annie!” Brain said and got an old oil lamp out of his bag.
    Annie ________NOT/CAN__________ hide her surprise: “What’s this? And where did you buy it?”
    “A magic lamp. And I didn’t buy it. It _________FIND_________ in the attic of an old country house a year ago. My friend found it when they _________REPAIR_________ the house. He says if you light the lamp and make a wish, the wish _________COME_________ true. It works particularly well with reasonable wishes that are not too big for the lamp. Its magic is not very strong.”
    Annie laughed aloud.  
    “I’ve never got a present _________ORIGINAL_________ than this. Ok, let’s test your lamp.” 
    Annie lit it and said: “I wish the whole class would come to _________I_________ party today”. As soon as she ________SAY__________ it, the doorbell rang impatiently.

  1. Recently my neighbour Michael Wilkins got a job in sales.  However, soon he found out that being a sales _________MANAGE_________ did not make him happy. He had to work long hours, and the job was _________STRESS_________ and demanding. The atmosphere in the office was very _________FRIENDLY_________ and the workers often quarreled with each other. So Michael decided to look for a more ________ATTRACT__________ job. In the local newspaper, he saw an _________ADVERTISE_________ from a pizzeria. It was close to his house and they needed a waiter. Michael decided to apply and didn’t regret it.
    “It’s good to feed people. I enjoy seeing them smile –  it’s important, isn’t it?” Michael says. 
    Next week he’s going to take a cooking course. On completing it, he will be able to make _________TASTE_________ pizzas himself! 

  1. School was finally over and I was about to enjoy my holidays. I was going to stay with my Aunt Helen, who lived in California. I thought that visiting _________SHE_________ would be great fun, but I felt nervous and scared. When my mum _________DRIVE_________ me to the airport, she noticed that something was wrong with me. 
    “Are you all right?’ she asked. “You’re afraid of flying, aren’t you?”
    “I _________NOT/WANT_________ you to notice. Yes, I’m a bit scared. Sorry.”
    “There _________BE_________ nothing to be sorry about! It’s natural to feel nervous before your first flight. I wish I ________CAN_________ go with you, but you know that I can’t.”
    It was true. My mum was the _________BUSY_________ woman in the world.
    “I’m fine, Mum. Don’t worry.” I tried to sound calm. “I ________CALL__________  you as soon as we land.”
    The plane _________TAKE_________ off smoothly. The pilot informed the passengers about the details of the flight. Then we _________OFFER_________ refreshing drinks.  I had some orange juice and closed my eyes.

  1. Father's Day in the UK is on the third Sunday in June. We enjoy this day very much and always arrange a _________CELEBRATE_________ at home. Our dad deserves it –  he is the most _________AMAZE_________ person I’ve ever met. He works as a train _________DRIVE_________ and he loves his job, though it takes time and energy. He cannot spend lots of time with us, but when he is at home, we have a _________FANTASY_________ time all together. Dad is a very _________CREATE_________ person and often invents new games for us. There are three children in our family and we always prepare a present for our dad together. Last time it was a large, _________COLOUR_________ book about the history of trains. The book starts with the invention of the first train and tells the whole story of the railway up to the present day.

  1. When I was at school, I spent two summers at my grandfather’s. They were the _________GOOD_________ holidays I’ve ever had. Grandpa lived alone in a country cottage near a small river. Grandpa’s hobby was making birdhouses. He always said that he _________NEED_________ a hobby to fill his free time.  Grandpa’s workshop was full of all kinds of tools. There _________BE_________ a big table, hammers, nails, paints, and what not. And all the tools _________KEEP_________ carefully in their places. That was Grandpa’s number one rule. The _________TWO_________ rule was, “Never buy a new tool if the old one still works.” In the workshop, when he _________MAKE_________ the birdhouses, I asked him lots of questions about birds. He _________KNOW_________ a lot about them!  He could tell one bird from another by their songs and footprints. I was surprised that Grandpa _________NOT/PAINT_________ the birdhouses in bright colours. He explained to ________I__________ that birds prefer natural colours that make their houses more difficult to notice and, for that reason, safer. 

  1. When I was at school, I spent two summers at my grandfather’s. They were the _________GOOD_________ holidays I’ve ever had. Grandpa lived alone in a country cottage near a small river. Grandpa’s hobby was making birdhouses. He always said that he _________NEED_________ a hobby to fill his free time.  Grandpa’s workshop was full of all kinds of tools. There _________BE_________ a big table, hammers, nails, paints, and what not. And all the tools _________KEEP_________ carefully in their places. That was Grandpa’s number one rule. The _________TWO_________ rule was, “Never buy a new tool if the old one still works.” In the workshop, when he _________MAKE_________ the birdhouses, I asked him lots of questions about birds. He _________KNOW_________ a lot about them!  He could tell one bird from another by their songs and footprints. I was surprised that Grandpa _________NOT/PAINT_________ the birdhouses in bright colours. He explained to ________I__________ that birds prefer natural colours that make their houses more difficult to notice and, for that reason, safer. 

  1. Sam went to the giant aquarium near his house at least three times a week. He liked the fish and crabs, but the most _________WONDER_________ creatures there were sharks. Sam could watch them for hours. He took photos through the glass and collected _________INFORM_________ about them. It surprised him how fast a massive shark could be. Before the aquarium opened, Sam thought it was ________POSSIBLE__________ for sharks to be so quick and graceful. He knew that sharks were _________DANGER_________ but they seldom attacked people. Many people, however, had hunted them and sharks had become endangered. _________FORTUNATE_________, now there were laws protecting sharks. In the future, Sam wanted to be a _________SCIENCE_________  and study sharks in the wild.

  1. The Grand National is a horse race which is held every year in Liverpool.  This _________COMPETE_________ involves a four-mile race, during which the horses have to go over 30 fences. The most _________FAME_________ horse in the Grand National was called 'Red Rum'. He was the _________WIN_________ of the race three times during the 1970s. The race is not only a sporting event but a _________FASHION_________ social event too. Men and women wear beautiful clothes, and the tickets to the race are very ________EXPENSE__________. The Grand National is broadcast all over the world and around 500 to 600 million people watch it.  However, though there are lots of horse racing fans, many people think it’s _________FAIR_________ and cruel to make the animals race and jump for people’s entertainment. 

  1. Dylan had a very old car. It was 20 years old.  But the faded paint made it look even _________OLD_________ than that. It _________BE_________ a shame to drive a car that looked like that and Dylan took it to a paint shop. There the car _________PAINT_________ dark blue for $200. Dylan was very pleased with the new look. The only problem was that the car stereo _________NOT/WORK_________ properly. Dylan managed to fix it himself. Then one day Dylan had a new problem –  petrol. He could smell it while he _________DRIVE_________ his car. Dylan asked himself “Am I driving a bomb? Could the car explode at any minute?” He knew the car manual by heart. He had car problems at least once a month, and he always found a way to fix _________THEY_________ himself. However, this time, it was a job for a mechanic. So he _________TAKE_________ his car to the closest service station. The mechanic said they _________TEST_________ the car to find out what the problem was. He told Dylan to come back in an hour. When Dylan came back, the mechanic said, "The problem is with the fuel pump. You need a new one. It costs $400." 
    "Let me think about it. I _________GIVE_________ you a call when I decide what to do."
    But Dylan had already decided what to do.

  1. I’ve worked in an office for many years. It’s a very _________STRESS_________ job and recently I’ve taken up a new hobby - gardening. My grandmother was a very good _________GARDEN_________. She never did gardening as a job but for pleasure and ________ENTERTAIN__________. She had the so-called “green fingers” - the flowers and fruit trees she planted grew very _________QUICK_________.  I wish I were as skillful as she was. It’s a pity my family does not share my interest in gardening. My children _________LIKE_________ working outdoors.  They prefer using the computer indoors. I don’t argue with them. Arguing and quarrelling makes me feel _________NERVE_________ and unhappy but cannot change the situation. I hope one day they will join me in the garden.

  1. Choosing a career is one of the most difficult choices we have to make in our lives. There are lots of interesting and _________EXCITE_________ professions, but which of them is the right choice for you? Researchers who work in _________EDUCATE_________ and sociology asked people across the UK what their dream job would be. And the results were quite _________EXPECTED_________. According to the survey, people find modest jobs like charity work or gardening the most _________ATTRACT_________. Often at the top of their list are jobs such as a nurse or a _________TEACH_________. The study also shows that people in the UK are very _________PRACTICE_________ and focus on dream jobs they have a realistic chance of getting. 

  1. Jenny doesn’t have many friends, but she has lots of books. Jenny likes fantasy stories best. She has a rich ________IMAGINE__________ which takes her to magical lands. Jenny often feels like she is a part of the story. She enjoys being in someone else's shoes, living in an _________ABSOLUTE_________ different world. When reading, Jenny may turn into a princess or become a mountain _________EXPLORE_________. She can see _________FANTASY_________ creatures and amazing places with her own eyes. Books are ideal friends for Jenny. They are entertaining and helpful. They can give her some _________PRACTICE_________ advice when she needs it and keep quiet when she doesn’t want to communicate. Books never _________AGREE_________ with her and they never argue. 

  1. Lots of companies do business online. They sell goods and services, or provide _________INFORM_________ to the general public. Doing business online is less _________EXPENSE_________ and more convenient than running a traditional business, with offices and lots of employees. Surprisingly enough, there are lots of very young people among online businessmen. In spite of their age, they are quite _________SUCCESS_________ and professional. For example, Ben Collins started his own online company when he was _________REAL_________ young. He was only 17. He says, “My business is ________NATIONAL__________ - we work with people from different countries. In a real office, some people would _________AGREE_________ that someone of my age could be a boss, but when I work online, nobody cares how old I am.” 

  1. Veronica loved autumn. It was her favourite season. There were many good things about it. She loved the smells, the changing _________LEAF_________, and the cool breezes. There _________BE_________ also apple pies, apple doughnuts and pumpkin pies. For Veronica, pumpkin pies were the _________GOOD_________, especially when her mother cooked them. Veronica had never tried to make a pie _________SHE_________, but she knew how, as she had seen how her mother made it many times. One day, on her way from school, Veronica _________BUY_________ a pumpkin and decided to cook it. She tried to do everything the same way her mother usually did. Veronica was in the kitchen and _________NOT/HEAR_________ the door bell. Her mother had to use her key to enter the house. 
    “Who _________MAKE_________ a pie?” her mother asked. “I can smell it. How soon is it going to be ready? I’m very hungry.”
    When the pie was ready, Veronica put it on the table. The pie looked nice: it _________DECORATE_________ with nuts and coloured sugar. Veronica’s mother had a large piece. She smiled and said that she _________LIKE_________ it very much. 



Предварительный просмотр:

Variant 1

1. How many lessons do you  usually have?

2. what subject do you find the most difficult?

3. what is your favorite weak day? Why do you like it?

4. what sports facilities do you have at school?

5. do you think a school uniform is necessary or not? Why do you think so?

6. what would you recommend your friend to improve English?

Variant 2      

  1. How often do you go to the cinema?
  2. What kind of films do you like most?
  3. Why do many people prefer watching films at home?
  4. What do you like you do in your free time?
  5. How do the films help you to improve English?
  6. What film do you recommend your friend to see and why?

Variant 3

 1. Who  usually does shopping in your family?

2. What can you buy in your nearest shopping centre?

3. How often do you go shopping?

4. Why are shopping centers popular with people now?

5. Why do some people hate shopping?

6. What time would you recommend busy people to do shopping?

Variant 4

  1. What kind of books do you like to read?
  2. Who is your favorite writer?
  3. How often do you borrow books from the library?
  4. Why do you think teenagers are reading less and less now?
  5. How do English books help to improve English?
  6. What book can you recommend your friend to read for pleasure?

Variant 5

  1. How much free time do you have on week days?
  2. How do you and your friends spend your free time?
  3. Who do you prefer to spend your free time with?
  4. Do you think everyone should find some time for doing sports? Why?
  5. What after school activities do you take part in?
  6. What hobby would you do in more free time?

Variant 6

  1. What is your favorite season?
  2. Where do you usually spend your summer holidays?
  3. What do you like to do during your winter holidays?

  1. How often do you and your friend travel?

  1. What type of transport is the most convenient? Why?

  1. What place can you recommend your foreign friends to visit? Why?

Variant 7

  1. Where would you like to live: in a big city or in a country?
  2. What are advantages for living in a country?
  3. What is the main ecological problem in the place you live?
  4. Do you and your friends care about ecological problems? Why?
  5. Have you ever taken part in an ecological project at school?
  6. What can you recommend to improve ecological situation in your hometown?

Variant 8

  1. How many lessons of PE (physical education) do you have a week?
  2. What sports facilities do you have in your school?
  3. What sport do you do regularly?
  4. What winter sports are popular with your friends?
  5. Would you like to do extreme sports? Why or why not?
  6. What would you recommend the teenagers to be healthy?

Variant 9

  1. When do you get up on weekdays?
  2. What do you prefer to eat for breakfast in the morning?
  3. How long does it take you to get to school?
  4. What is your favorite part of the day: morning, afternoon or evening? Why?
  5. What do you do to help your parents about the house?
  6. What would you recommend to you friend who is often late for school?

Variant 10

  1. How old are you?
  2. When did you get your first mobile phone?
  3. What do you usually do with your mobile phone?
  4. How do you feel when you forget your mobile phone?
  5. Do you think it is right that it is not allowed in some schools?
  6. In what places would you recommend your friend to  switch off or turned out your mobile phone

Variant 11

  1. How old are you?
  2. How much time do you spend in the Internet every day?
  3. What do you use the Internet for?
  4. What do you do at you IT or study lessons?
  5. Why are computer skills important for everyone?
  6. What would you recommend to a person who spend too much time on the Internet?

Variant 12

  1. What is your favorite school subject?
  2. When did you start to learn English?
  3. What do you do at your English lessons?
  4. Do you find learning English easy or difficult? Why?
  5. What other foreign language would you like to learn?
  6. What would you recommend a person  do to improve English?

Variant 13

  1. What do you enjoy doing in your free time?
  2. How often do you go to the cinema or to the theatre?
  3. How much time do you usually watch TV every day?
  4. What TV programmers are popular in your family?
  5. Do you prefer TV or the Internet? Why?
  6. What TV program would you recommend your friend watch?

Variant 14

  1. How long have been living in your city, town or village?
  2. What is place you live famous for?
  3. When was city or town founded?
  4. What season is the best to visit your village or town? Why?
  5. What is your favorite place in your city, town or village? Why do you like it?
  6. What place of your hometown can you recommend visiting? Why?

Variant 15

  1. What foreign language do you learn in your school?
  2. How many English lessons a week do you have?
  3. For what reason do people learn foreign languages nowadays?
  4. What is the most effective method of learning foreign language in you opinion?
  5. Do you think English will be useful in your future? In what way?
  6. What language would you recommend you English-speaking friend to learn? Why?

Variant 16

  1. What time do you usually go to bed in the evening?
  2. How much time do you spend doing your homework?
  3. What sport do you do regularly?
  4. What do you do to help your parents about the house?
  5. How do you spend your weekends?
  6. What would you like to change in your daily routine?

Variant 17

  1. When do you start your day?
  2. How many classes do you usually have?
  3. What sport facilities are there in your school?
  4. What is your school uniform like?
  5. What school events  do you have during a school year?
  6. What would you like to improve at your school?

Variant 18

  1. How old are you?
  2. How many times a week do you do sports?
  3. What sport is the most popular with teenagers in your region?
  4. What sport facilities are available in the place you live?
  5. Why do you think it is important to keep fit?
  6. What would you advise to a person who wants to keep fit?



Предварительный просмотр:

Вариант №1

You have 30 minutes to do this task.

You have received a letter from your English–speaking pen friend, Ben.

…We’ve moved house and now I have to go to a new school. It’s a pity because I liked my old school and I had a lot of friends there. Now I feel a bit lonely but I hope to find some friends here too….

…What do you like most about your school? Have you got a lot of friends at school? What do you usually do with your school friends in your free time? …

Write him a letter and answer his 3 questions.

Write 100120 words. Remember the rules of letter writing.

Вариант № 2

You have 30 minutes to do this task.

You have received a letter from your English-speaking pen friend, Ben.

…I am so fond of Japanese cartoons, that I’ve decided to learn Japanese. Mum says it’s hard, but I think it’s worth it. Besides, their writing is cool! …

…What languages do you learn at school? What other language would you like to learn, why? What do you think about my choice of Japanese? …

Write him a letter and answer his 3 questions.

Write 100120 words. Remember the rules of letter writing.

Вариант № 3

You have 30 minutes to do this task.

You have received a letter from your English–speaking pen friend, Ben.

…I am a bit sad because I have just quarrelled with my mum. I didn’t want to wash the dishes and make my bed because there was an interesting film on TV…

…Do you think children should help their parents with the housework, why/why not? What do you do about the house? What do you usually do in your free time? …

Write him a letter and answer his 3 questions.

Write 100120 words. Remember the rules of letter writing.

Вариант № 4

You have 30 minutes to do this task.

You have received a letter from your English-speaking pen friend, Ben.

…My cat is getting old. But he’s still able to jump on the fridge and he likes to watch Mum making dinner for him. And he is so cool: big, white and furry! He is such fun! I can’t understand people who prefer to live without pets…

…Why don’t some people want to have any pets? Do you approve of keeping exotic pets like spiders or snakes, why? What pets are popular in your country? …

Write him a letter and answer his 3 questions.

Write 100120 words. Remember the rules of letter writing.

Вариант №5

You have 30 minutes to do this task.

You have received a letter from your English-speaking pen friend, Ben.

…Sometimes I hate our school radio because they talk about things everyone knows. And news should be fresh! One day I will be a reporter and will show them how to produce really good news…

…How do you get to know what’s happening in the world? What are you going to be? Do you think English will be necessary for your future career, why? …

Write him a letter and answer his 3 questions.

Write 100120 words. Remember the rules of letter writing.

 Вариант №6

ou have 30 minutes to do this task.

You have received a letter from your English–speaking pen friend, Ben.

… I can’t imagine my life without my computer and the Internet! Now I can write all my stories and immediately send them to my friends to read...

…What do you use your computer for? How much time do you spend working on your computer? What else do you do in your free time? …

Write him a letter and answer his 3 questions.

Write 100120 words. Remember the rules of letter writing.

Вариант №7

You have 30 minutes to do this task.

You have received a letter from your English-speaking pen friend, Ben.

…and then she gave me a present: two tickets to the theatre. It was a great performance and the actors were wonderful. I am not a theatre goer but even I was impressed. …

…Do you think it’s exciting to be a famous actor, why? When did you last see a theatrical performance? What do you like to do in your free time? …

Write him a letter and answer his 3 questions.

Write 100120 words. Remember the rules of letter writing.

Вариант №8

You have 30 minutes to do this task.

You have received a letter from your English-speaking pen friend, Ann.

…Oh, my friend is so charming! Everybody in the class is ready to help her. But, you know, I sometimes think she’s just using people to get what she wants…

…What kind of person is your best friend? When and where did you meet? How do you spend time together?…

Write her a letter and answer her 3 questions.

Write 100120 words. Remember the rules of letter writing.

Вариант №9

You have 30 minutes to do this task.

You have received a letter from your English-speaking pen friend, Ben.

…What an awful task  to read boring books all summer! And the list is so long! I like reading but the books I like are usually not my school’s choice…

…How long is your school’s list of books? What kind of books do you like to read? Do you think an electronic book is a good choice when you’re travelling in the summer, why? …

Write him a letter and answer his 3 questions.

Write 100120 words. Remember the rules of letter writing.

Вариант №10

You have 30 minutes to do this task.

You have received a letter from your English-speaking pen friend, Ben.

…Yesterday was a special day  we had the Earth Hour. My town (and lots of towns around) went dark for one hour. It’s one of the ideas of an ecological group called the Green Movement. We switched off the lights to draw people’s attention to the poor ecological situation on the planet …

…Do people have the Earth Hour in your country? What can people do to save energy in their everyday life? Do you approve of the Green Movement, why? …

Write him a letter and answer his 3 questions.

Write 100120 words. Remember the rules of letter writing.

Вариант №11

You have 30 minutes to do this task.

You have received a letter from your English-speaking pen friend, Ben.

…. My older brother has decided to collect mugs, imagine that. He has already got eight mugs from different places he has visited. It’s a nice collection but it takes up so much room …

 …Why do people collect things? What do you collect or what would you like to collect? What other hobbies are popular with teenagers in your country? …

Write him a letter and answer his 3 questions.

Write 100120 words. Remember the rules of letter writing.

Вариант №12

You have 30 minutes to do this task.

You have received a letter from your English–speaking pen friend, Ben.

… I’m having a very busy time now as I have to get ready for my exams. As far as I know, all students in Russia have to take school exams too…

…When are you going to have your exams? What exams have you chosen and why? What are your plans for the summer holidays? …

Write him a letter and answer his 3 questions.

Write 100120 words. Remember the rules of letter writing.

Вариант №13

You have 30 minutes to do this task.

You have received a letter from your English–speaking pen friend, Ben.

… Today is Sunday, but I feel bored and unhappy. It’s already been raining hard for two days and there’s nothing to do at home…

…What is your favourite season and why? What do you do when the weather is rainy? How do you usually spend your weekends? …

Write him a letter and answer his 3 questions.

Write 100120 words. Remember the rules of letter writing.

Вариант №14

You have 30 minutes to do this task.

You have received a letter from your English-speaking pen friend, Ann.

…My friend has decided to become a model and has just gone on a diet. I told her that it could be dangerous but she wouldn’t listen. Besides, she is a vegetarian, so there are not many products she can let herself eat now…

…Why do people become vegetarians? What do you do to keep fit? What food do you try to avoid? …

Write her a letter and answer her 3 questions.

Write 100120 words. Remember the rules of letter writing.

Вариант №15

You have 30 minutes to do this task.

You have received a letter from your English–speaking pen friend, Ben.

...I’ve just joined our school Drama club. We give performances in English and in French. Our parents and classmates think that our performances are very good. After all, school is not only about learning...

…What do you like most about your school? What school clubs do you have? Have you joined any of your school clubs, why? …

Write him a letter and answer his 3 questions.

Write 100120 words. Remember the rules of letter writing.

Вариант №16

You have 30 minutes to do this task.

You have received a letter from your English-speaking pen friend, Ben.

…Yesterday I did a test to see which job is the most appropriate for me. According to the results I should become a doctor. But it would be absolutely impossible because I am afraid of blood…

…What future career would you like to have, why? Do your parents agree with your choice? In what way will English be useful for your career? …

Write him a letter and answer his 3 questions.

Write 100120 words. Remember the rules of letter writing.

Вариант №17

You have 30 minutes to do this task.

You have received a letter from your English–speaking pen friend, Ben.

...What a journey! We went by car to Brighton and spent a couple of days at the seaside. I wish we could go there again! ...

…Why is travelling so popular? Where would you like to go? How do you prefer to travel and why? …

Write him a letter and answer his 3 questions.

Write 100120 words. Remember the rules of letter writing.

Вариант №18

You have 30 minutes to do this task.

You have received a letter from your English-speaking pen friend, Ben.

… My mum always says “There’s no bad weather, there’re bad clothes!” That’s why she keeps telling me to put on warm clothes even in spring. …

…Do you agree with the English proverb about bad weather and bad clothes, why? What clothes do you prefer to wear out of school? What are your favourite colours for clothes? …

Write him a letter and answer his 3 questions.

Write 100120 words. Remember the rules of letter writing.

 Вариант №19

You have 30 minutes to do this task.

You have received a letter from your English-speaking pen friend, Ben.

…Yesterday our teacher asked us to stay after classes. A basketball star came to us to talk about sport in everyday life. The way he spoke about being a teenager was funny. He said he didn’t like getting up early!…

…When do you get up in the morning? What do you do to keep fit? Would you like to go in for sports professionally, why? …

Write him a letter and answer his 3 questions.

Write 100120 words. Remember the rules of letter writing.

Вариант №20

You have 30 minutes to do this task.

You have received a letter from your English-speaking pen friend, Ben.

…I’ve just been to the zoo. What lovely animals are there! Bears, tigers and even sharks! When I look at them in the zoo I can hardly imagine that they could be dangerous or eat other animals…

… What do you think is better for animals  to live in the zoo or in the wild, why? What animals are common in your region? Have you ever seen any animal in the wild? …

Write him a letter and answer his 3 questions.

Write 100120 words. Remember the rules of letter writing.

 Вариант №21

You have 30 minutes to do this task.

You have received a letter from your English–speaking pen friend, Ben.

...One of my friends gets ill very often. The doctor says that it’s because he lives in a big city with a poor environment...

…What ecological problems are the most serious in your country? What can teenagers do to make their hometown cleaner? Would you like to live in a big city or in the country, why? …

Write him a letter and answer his 3 questions.

Write 100120 words. Remember the rules of letter writing.

Вариант №22

You have 30 minutes to do this task.

You have received a letter from your English–speaking pen friend, Ben.

...My friend was very impressed by the Moscow Kremlin. He said that Russian architecture is fascinating.  I'd like to go to Russia for my holidays, too...

…How do you usually spend your holidays? What is the best season for travelling in Russia, why? What tourist attractions would you recommend seeing in your country? …

Write him a letter and answer his 3 questions.

Write 100120 words. Remember the rules of letter writing.

Вариант №23

You have 30 minutes to do this task.

You have received a letter from your English-speaking pen friend, Ann.

… My older brother has got a new hobby  he has become a mountain biker. I know that mountain biking is a dangerous kind of sport and I worry about him. I would like him to spend more time at home. I wish he had chosen a safer entertainment…

…What indoor sports are most popular with teenagers in your country? Why do teenagers often do extreme sports? What kind of sport is your favourite? …

Write her a letter and answer her 3 questions.

Write 100120 words. Remember the rules of letter writing.

Вариант №24

You have 30 minutes to do this task.

You have received a letter from your English-speaking pen friend, Ben.

… Last year I started learning Russian. It’s a very difficult language but I’m trying my best. I’d like to come to Russia one day to practise it. …

…Which season would you recommend for the trip? What places are worth seeing in Russia? What country would you like to visit, why? …

Write him a letter and answer his 3 questions–

Write 100120 words. Remember the rules of letter writing.

 



Предварительный просмотр:

                                                         ВАРИАНТ № 1 (9 КЛАСС - 2016)

Прочитайте тексты и установите соответствие между текстами АE и заголовками 16. Запишите свои ответы в таблицу. Используйте каждую цифру только один раз. В задании есть один лишний заголовок. 

1. 

Why did people start the postal service?

2. 

What is snail mail?

3. 

Where did the first delivery system appear?

   

4. 

What are the advantages of e-mail?

5. 

What was the first stamp like?

6. 

How were the letters paid for?

A. 

For as long as humans have existed there has been a need to keep in touch and to transfer important information between people in different places. Before the invention of writing, oral messages were carried from one person to another between towns. Writing made it much easier to send longer messages; however, it was still difficult to make sure that your message got to the right place.

B. 

It was used by the Roman officials to transfer information throughout the Empire. Staging posts and a system with horses and carriages meant that messages could move quickly, by using many riders instead of one. It was very important for business and military reasons that good communication system existed. However, the Romans were not the first to realize this. The Chinese and Persian empires used systems of horses and riders more than 500 years before the Romans.

C. 

Before the invention of the postage stamps, letters were “franked”. It was marked on the letter that delivery had been paid for. This could have been either written or stamped. A post-mark was also stamped on the letter. Invented in 1660 in England, this was a mark that showed where and when the letter had been posted. It was used to see how long it took to deliver the letter  to make sure the service was reliable.

 

D. 

It is the humorous term used by e-mail users for the old-fashioned letters-in-envelopes postal system. It means that such letters travel very slowly, which actually is rather unfair. In Britain you can send letters first or second class; the first class ones normally get to their destination, anywhere in the country, the next morning. The postal service is called the Royal mail, and all the British stamps have the head of the Queen.

 

E. 

They were invented in Great Britain. It was a British man called Rowland Hill who proposed a stamp to be stuck on the letter to identify that postage had been paid. The first stamp was issued in 1840. It was called the Penny Black and the profile of Queen Victoria’s head was depicted there. The stamp cost 1 pence and was darkly coloured. About 65 million Penny Blacks were issued, and nowadays it is not a very rare stamp.

                                                ВАРИАНТ № 2 (9 КЛАСС - 2016)

Прочитайте тексты и установите соответствие между текстами АE и заголовками 16. Запишите свои ответы в таблицу. Используйте каждую цифру только один раз. В задании есть один лишний заголовок.

 

1. 

Languages spoken in Britain

2. 

The origin of English

3. 

Modern language tendencies

   

4. 

Regional varieties of English

5. 

English in Australia

6. 

Global English

A. 

English is basically a Germanic language with a lot of Latin words in it. In simple terms, that means that the grammar and many of the most frequent words are Germanic, and the more formal or technical vocabulary is Latinate. The linguistic mixture is a result of historical events. But the simple historical facts appear not to explain everything about the development of the language. One interesting question is why the British did not learn Latin from the Romans.

B. 

It came as a surprise to many people when a survey showed that 172 languages were spoken by children in London schools: Chinese, Turkish, Italian, Spanish, Punjabi and others. Some of these, like the West African language Ga, only have a couple of speakers. But others, like Punjabi, are quite significant linguistic community, with their own radio programmes and newspapers, and classes for children – to ensure that they don’t forget the language of their grandparents.

C. 

A nasty shock awaits many visitors to Britain. Imagine you have learnt English for years, you can read newspapers and you have no problem following the television, but when you go into a shop in Newcastle you can’t understand a word they are saying. It is the accent, mostly the vowels, which gives the visitor a problem in the shop. Some accents are so strong that they present problems for British people, too. Intonation patterns also differ between regions.

 

D. 

Billions of people speak English, two-thirds of the planet’s scientists write in English, and over 80% of the world’s electronic information is stored in English. But not all the British are really self-satisfied about the status of their language. But they are quite well aware that today it is out of their hands: the reasons for the popularity of English are either lost in history, or something to do with the superpower on the other side of the Atlantic.

 

E. 

Let’s look at the speech of young people in Britain. Here we can find several interesting developments. One is a spread of a light London accent over much of the country. Another is an openness, through the media, to American and Australian influences. The Australian effect is quite recent, and the results from the huge popularity of Australian TV soap operas. It is the phrases, idioms and grammatical forms which are catching.

ВАРИАНТ № 3 (9 КЛАСС - 2016)

Прочитайте тексты и установите соответствие между текстами АE и заголовками 16. Запишите свои ответы в таблицу. Используйте каждую цифру только один раз. В задании есть один лишний заголовок.

 

1. 

Early mobile phones

2. 

Annoying mobiles

3. 

Expensive mobiles

   

4. 

Fashionable mobiles

5. 

Multifunctional mobiles

6. 

Modern mobile phones

A. 

Lars Magnus Ericsson, was the first man who had a telephone in his car. When he saw a place where he could get to phone lines, he connected to them with a pair of long electric wires. Then in Russia two Soviet engineers successfully tested a mobile phone installed in a car. It could connect to a local telephone network within 20 kilometers. A year later, two engineers from Europe tried to use antenna in the phone.

B. 

The company of mobile phones in Finland started to sell their products in 1970. There were several types of phones: 2G and later 3G. Third-generation phones are now used everywhere in the world. They are small, flat and very comfortable to use. Some people say that the only problem with the new phones is to remember all the functions. The mobile phone is often called the Seventh of the Mass Media (with Print, Recordings, Cinema, Radio, TV and Internet the first six).

C. 

In many countries, most adults and many children now have mobile phones. Mobile phones are used for a variety of purposes, including keeping in touch with the family, running business, and for emergency calls. Children and adults often play mobile phone games or use the phone as an audio player. In Japan, phone companies provide immediate notice of earthquakes and other natural disasters to their customers free of charge. In the event of an emergency, disaster response crews can find injured people using the signals from their mobile phones.

D. 

Today’s mobile phones do more than just offer voice, email, Web and music services. They are stylish accessories, too. Cool design has always played a great role in digital business. People, young people especially, use the phone to express their self. Just like clothes, phones can carry a message of sports lifestyle, luxury, adventure or romance. Mobile phone companies introduce their new collections every season and it’s rather difficult to keep up with today’s mobiles.

E. 

Mobile phones have increased greatly in the world. Some people carry more than one mobile phone for different purposes, such as for business and personal use. The mobiles are to be seen absolutely everywhere – in schools, restaurants, theatres and even churches. They ring during lectures, meetings and in classical music concerts. In recent survey, 62 % of people said that the most irritating thing in their lives was mobile phones on the train!

ВАРИАНТ № 4 (9 КЛАСС - 2016)

Прочитайте тексты и установите соответствие между текстами АE и заголовками 16. Запишите свои ответы в таблицу. Используйте каждую цифру только один раз. В задании есть один лишний заголовок.

 

1. 

Geography

2. 

Famous people

3. 

Population

   

4. 

Education

5. 

Sports

6. 

Holidays

A. 

The United States is the second largest country in North America after Canada. It is washed by the Atlantic Ocean in the east, by the Pacific Ocean in the west and by the Gulf of Mexico in the south. The country is so big that there are four standard time zones. It also possesses many islands in the Pacific Ocean and Puerto Rica and the Virgin Islands in the Atlantic. The USA has the third largest population in the world (after China and India).

B. 

The American Dream, the hope for a better life in a new country, has attracted people from all over the world. Most of them leave their homeland because they are poor and have few opportunities. Some are forced to leave because of famines and civil war; others come for religious freedom. There are nearly 250 million people living in the USA. 10% are Spanish-speaking people than people of Asian and Pacific Island origin.

C. 

Each September, more than fifty million students between the ages of five and seventeen start the new school year. Students attend classes for an average of five hours a day, five days a week, and they have homework assignments to complete after school and on weekend. Many students sign up for after-school programs at their school or nearby. Sports are very popular. There are children’s baseball, soccer, football, and basketball teams in almost every school. 41% of high school graduates go to college.

D. 

This day celebrates Christopher Columbus’s arrival in the Americas in 1492. As Columbus was Italian, working for Spain, Columbus Day is especially important for many Italian-Americans and Hispanic-Americans. Since 1971, the holiday has been commemorated in the U.S. on the second Monday in October. Officially, the people of the USA are invited to celebrate the anniversary of the discovery of their country with church services and other activities. In some towns and cities, special church services, parades and large events are held.

E. 

Most Americans enjoy sports – both playing sports themselves and watching their favourite teams and competitions. Children are encouraged to participate in sports at an early age. The rivalry between high schools and colleges in sporting events goes right through the school year from football in September to track and field in June. Sports are important in college. Students who show talent in a sport like tennis, track and field, swimming, football, or basketball can apply for sports scholarships.

ВАРИАНТ № 5 (9 КЛАСС - 2016)

Прочитайте тексты и установите соответствие между текстами АE и заголовками 16. Запишите свои ответы в таблицу. Используйте каждую цифру только один раз. В задании есть один лишний заголовок.

 

1. 

Name meaning

2. 

Pumpkin brief facts

3. 

Cooking with pumpkin

   

4. 

Activities involving pumpkin

5. 

Pumpkin’s recipes

6. 

Pumpkin’s magical popularity

A. 

Pumpkin is a large round fruit with a thick orange skin and large seeds. Pumpkins are 90% water but they contain potassium and vitamin A. Six of seven continents can grow pumpkins. Nowadays pumpkins have come to symbolize not only the lovely color of the autumn but the spirit of the season as well. A sizable number of movies were made with pumpkin and Halloween themes. Most memorable, perhaps, is the pumpkin carriage in Cinderella.

B. 

Pumpkins are believed to have originated in North America. Native American Indians ate pumpkins centuries before the Europeans landed. Seeds from related plants have been found in Mexico dating back to 7000 to 5500 B.C. References to pumpkins date back many centuries. The word pumpkin originated from the Greek word for "large melon" which is "pepon." "Pepon" was changed by the French into "pompon." The English changed "pompon" to "Pumpion."

C. 

In the United States, pumpkins go hand in hand with the autumn holidays of Thanksgiving, and, of course, Halloween. At almost every Thanksgiving table is the customary 'Pumpkin Pie'. Pumpkins are carved into decorated lanterns for Halloween. Pumpkin is one of the important symbols of the harvest festival and has been an American-favourite for over 400 years now. A lot of autumn festivals are dedicated to the pumpkin. Farmers from all over the US compete to determine who has grown the heaviest pumpkin.

 

D. 

In the USA, pumpkin is a very popular Thanksgiving dish. It is not sure whether pumpkin was one of the dishes in the first thanksgiving dinner. But they were, however, a part of all traditional meals long before the arrival of the first Europeans. Most parts of the pumpkin are suitable for eating, including the shell, the seeds, the leaves, and even the flowers. When ripe, the pumpkin can be boiled, baked, steamed, or roasted.

E. 

The pumpkin has been very important for centuries. It has been popular with wizards, witches and fairies. Maybe that’s why it is celebrated in so many festivals, folklore and fiction. People make jack o’lanterns at Halloween in the USA. In European and American fairy-tales witches often turn people into pumpkins. Even in modern The Harry Potter novels pumpkin juice is the favourite drink of the students of Hogwart’s school.

ВАРИАНТ № 6 (9 КЛАСС - 2016)

Прочитайте тексты и установите соответствие между текстами АE и заголовками 16. Запишите свои ответы в таблицу. Используйте каждую цифру только один раз. В задании есть один лишний заголовок.

 

1. 

Pages of History

2. 

A national holiday

3. 

Languages

   

4. 

Cities and towns

5. 

Geography

6. 

Economy

A. 

The island of Ireland is bounded to the north and west by the Atlantic Ocean, the east coast is washed by the Irish Sea. The western landscape mostly consists of cliffs, hills and mountains. The country's interior is relatively flat, traversed by rivers such as the River Shanon and numerous lakes. Agriculture and food is an important industry in Ireland, with about 64% of the total land area used for agriculture. The main cities are Dublin, Cork and Limerick.

 

B. 

Ireland has always been an agricultural country. It is almost self-sufficient in food and a major exporter. High technology industries have now made Ireland into Europe's largest (and the world second largest) exporter of computer software. American companies like Apple and Intel have invested heavily in Irish computer factories. The secret of Ireland's success is a young and highly educated population, who speak English. Now Dublin, the capital of the country, is an important world financial centre.

 

 

C. 

Saint Patrick is the patron saint of Ireland and his day is celebrated on the 17th March with parades, "wearing of the green," music and songs, Irish food and drink, and activities for kids such as crafts, coloring and games. It's a time for fun. On that day almost every town has a parade of bands and floats. Some floats depict scenes from Irish history, others advertise products. The largest parade is held in Dublin.

 

D. 

The Celts were the first known inhabitants of the island. They gave the Irish the basis of their language and culture. Saint Patrick, the patron saint of Ireland, is said to have brought Christianity to the Celts. Later, the Vikings arrived from Scandinavia and founded Ireland's two main cities, Dublin and Cork. At the beginning of the seventeenth century the British started an invasion on Ireland. They imposed their language and laws.

 

E. 

It was imposed on the people of Ireland, yet it has produced some of the most imaginative writers of English literature. This creativity goes back the Gaelic poetry of the Celts. Irish is still the official language of Ireland, but the great majority of people speak English. Most television programmes are in English too. As a result of immigration, Polish is one of the most widely spoken languages in Ireland after English and Irish.

ВАРИАНТ № 7 (9 КЛАСС - 2016)

Прочитайте тексты и установите соответствие между текстами АE и заголовками 16. Запишите свои ответы в таблицу. Используйте каждую цифру только один раз. В задании есть один лишний заголовок.

 

1. 

School rules

2. 

School subjects

3. 

After school activities

   

4. 

School Facilities

5. 

School Punishment

6. 

A typical English school

A. 

Chiswick School is a secondary school in London. It has about 1.250 pupils between the ages of 11 and 19. Like in most British schools, all the pupils study the same subjects in the first three years. Then they work for their GCSE exams – usually taking about nine subjects. Everyone takes English, Maths and Science, but there is quite a wide choice of other subjects. After the exams, some pupils leave and get jobs; others go on to do their A levels at other schools or further education college; some stay on in school.

B. 

There are six tennis courts, a gym, two squash courts and a large games field, where boys and girls play football and hockey. Pupils can also row on the nearby River Thames. There are also three main computer centres, and most subjects (such as Chemistry and Languages) have their own computers. All pupils study Information Technology. The school has a large library and a Learning Resources Centre with photocopying facilities, computer scanners and printers.

C. 

When the classes are over, Chiswick students are busy too. There is judo, dance, cooking and chess provided by the school at lunch time and in the afternoon. There are music and photography clubs. There is a theatre group which puts on a play at the end of each year. The teachers and the parents encourage as many pupils as possible to take part in numerous clubs. In the holidays, the school organizes trips such as camping in Wales or skiing in Switzerland.

 

D. 

There is one basic law in the school: to respect others and their property and behave in a normal, sensible way, with due consideration for the health and safety of all. In really serious cases, it is possible to exclude pupils for a period of time from school, or to expel them permanently. As in most schools in Britain, pupils have to wear a uniform. There is some resistance to wearing school uniform, but generally teachers and parents agree that the uniform is good for discipline.

E. 

If, as a pupil, you misbehave there is a system known as WRO – Warning, Report, Out. First you get a warning from your teacher, you have to carry a form around with you and the teacher signs it after each lesson – to show that you were present and behave well. If you do anything wrong at this stage, you are out; you have to go to the hall to join any other pupils in trouble. In the hall, you all work in silence under the supervision of a teacher.

ВАРИАНТ № 8 (9 КЛАСС - 2016)

Прочитайте тексты и установите соответствие между текстами АE и заголовками 16. Запишите свои ответы в таблицу. Используйте каждую цифру только один раз. В задании есть один лишний заголовок.

 

1. 

What do weather people do?

2. 

What does the name mean?

3. 

What helps predict the weather?

   

4. 

How was the thermometer invented?

5. 

What is the most beautiful weather phenomenon?

6. 

How does the thermometer work?

A. 

Since ancient times people all over the world have tried to understand such physical phenomena like rain, snow and wind. The Greek scientist and philosopher Aristotle wrote a book named Meteorologica, which was about the basic elements of the environment: the earth, the winds, the seas and rivers, and of course, the weather. He is considered to be the founder of a science, which studies the atmosphere, and the title of his book has become the name of the science.

 

B. 

Meteorology supposes a lot of observations, researches and analyses. Without them it is impossible to say what the weather will be like tomorrow. In 1654 one scientist started an ambitious research project. Its aim was to record weather patterns in different Italian and European cities. The received information was studied and analyzed by the scientist. Those results were very important for meteorology. So that project gave an opportunity for G. Fahrenheit to create the modern thermometer.

 

C. 

If you enjoy everything from watching clouds to reading about hurricanes and tornados, then maybe meteorology is the career for you. Meteorologists, can be found all over the world doing some very interesting things! They may study the ozone layer or air pollution or make weather observations in Antarctica. They might monitor rainfall and give river flood warnings or fly in aircraft investigating hurricanes. Weather is world-wide and so are people, who study the weather!

 

D. 

Have you ever wondered how people are able to predict the weather ahead of time? Wouldn't it be great to know if there is a chance of rain for your football game this weekend? Weather forecasting, a prediction what weather will be like tomorrow or next week, takes into consideration many aspects, but one of the most crucial weather features is the wind. By learning how the wind affects the weather, we can learn a lot about how to predict it.

 

E. 

When light and water meet in the sky on a summer day, a rainbow appears. Such a wonderful sight! The legends of many cultures see the rainbow as a kind of bridge between heaven and earth. But it's just sunlight and raindrops. When sunlight enters a drop of rain, a part of it does not pass directly through but is reflected from the inner surface and appears from the side from which it entered.

ВАРИАНТ № 9 (9 КЛАСС - 2016)

Прочитайте тексты и установите соответствие между текстами АE и заголовками 16. Запишите свои ответы в таблицу. Используйте каждую цифру только один раз. В задании есть один лишний заголовок.

 

1. 

Diverse population

2. 

City lifestyle

3. 

Media and communication

   

4. 

Countryside

5. 

Environmental issues

6. 

Leisure and pastime

A. 

The USA is often called a melting pot – a total mix of races and nationalities. In fact, the same could be said of Britain, except this process took far longer. The British today are the result of wave after wave of conquest, migration and mixing. Now there are many people of all colours and races in the UK. They are mostly former inhabitants of the former British colonies. There are special newspapers, magazines, radio and TV programmes for them.

B. 

Visitors to Britain are often surprised by the titles of the magazines on newsagents' shelves. Some of them are extremely specialized and lots of them are on new technology. The British are also great newspaper readers.  An almost universal British custom, going for a walk, is another surprising fact.

C. 

Recently people have been moving into suburbs, smaller towns and the country. Villages, farms and cottages, which used to be full of agricultural workers, are now the homes of people who drive to their offices in cities. Even so Britain remains one of the most urbanised countries in the world, with nine in ten of the population living in towns or cities.

 

D. 

The Cotwolds are gentle green hills in agricultural country to west of England. A mixture of small fields separated by old-dry stone walls and lines of trees make this a perfect example of traditional English rural area. Farming is balanced with cattle and sheep, and small spots of woods. Many farms, houses and churches, like the walls dividing the fields, are built of local honey-coloured stone. Villages with climbing roses around the doors of the cottages are small and beautifully kept.

E. 

The poet William Blake wrote 200 years ago of “England's green and pleasant land.” In spite of their mainly urban lives, this image is still strong in British people imagination. They love the countryside and watch hours of wildlife programmes on TV every week. Some of the first environmental organisations in the world started in Britain. The British managed to defeat the London smog: replacing coal by oil and gas. And they are rather proud of this achievement.

ВАРИАНТ № 10 (9 КЛАСС - 2016)

Прочитайте тексты и установите соответствие между текстами АE и заголовками 16. Запишите свои ответы в таблицу. Используйте каждую цифру только один раз. В задании есть один лишний заголовок.

 

1. 

Where to find traditional British food

2. 

Why fast food appeared

3. 

Modern British food

   

4. 

Traditional British drink

5. 

What they used to eat

6. 

What kind of tea they prefer

A. 

If you go back to the time of Queen Elizabeth (1558–1603), people really knew how to eat and Chefs used to travel round Europe to get new ideas and ingredients. It was also the time when British explorers sailed all over the world. They brought home all sorts of exotic food: rice and tea from China, spices from India, coffee from Arabia. In the Americas they found tomatoes, potatoes, peanuts, pineapples, sugar and vanilla.

B. 

In the past, without cookbooks and TV programmes, women learnt from their mothers and spend hours every day in the kitchen. But then, around the end of the 18th century, life in Britain changed dramatically. The industrial revolution took families from farms in the country to the crowded cities. Men and women worked long hours in the factories. So they had no longer time or energy to cook properly at home. There was already an interest in fast food.

 

C. 

Every country seems to have a national cuisine and to be proud of it. So what is Britain famous for? The sad truth is that most of the British gave up cooking a long time ago and started buying ready-made meals from the shops. They love tins, tubes, packets and frozen food of all kinds. Of course, convenience food is an international phenomenon, but it started earlier and has gone further in Britain than in most countries.

D. 

It’s more than just a tradition – it is a way of life. Many people have tea first with breakfast, then at mid-morning, with lunch, at tea-time (around 5 o’clock), with dinner, and finally just before bed. As a nation, they get through 185 million cups per day! Most British use tea bags these days, but serious tea lovers still go through an almost Japanese-style ceremony. The selection of tea is very personal.

E. 

The country has some excellent food: lamb from Wales, shellfish and salmon from Northern Ireland, fresh fish from Scotland, cheeses from England and Wales. Unfortunately, good British food is difficult to find. Only 2% of restaurants in London serve British food. There are many more Italian, Chinese and Indian restaurants. You can have a good “British” meal without spending a lot of money by going to a pub (or public houses) that you can see everywhere in Britain.

ВАРИАНТ № 11 (9 КЛАСС - 2016)

Прочитайте тексты и установите соответствие между текстами АE и заголовками 16. Запишите свои ответы в таблицу. Используйте каждую цифру только один раз. В задании есть один лишний заголовок.

 

1. 

Economy and industries

2. 

Food and drinks

3. 

Main cities

   

4. 

Leisure time

5. 

Population

6. 

Pages of history

A. 

Australia once had close ties with Great Britain. Until fifty years ago, almost all Australians were of British or Irish origin. But after World War II more than two million people arrived from Europe. New Australians, as they were known, were from Greece, Germany and Italy. Mass migration has changed Australia into a multicultural society. Today, people of Anglo-Irish origin are a minority. Many recent immigrants are from South East Asia, the Philippines and China .

B. 

Australia's prosperity has been founded on its natural resources. It produces almost every mineral from gold to uranium. Traditionally, Australia was a farming country and its wealth was founded on wool, cattle and wheat. It is still a major exporter of food, making it the breadbasket of Asia. Agriculture and mining, however, employ comparatively few people. Tourism is now Australia's largest industry, employing 6% of the population. More than three million visitors come to the country each year.

C. 

In spite of the importance of the bush to Australian folklore, 90% of Australians live in cities. Almost everyone lives within a few kilometres of the sea and of the ocean. The beaches are used in same way as parks: families have picnics, people of all ages jog, play volleyball, swim or just walk. There is great interest in water sports like water skiing, surfing, swimming and sailing. Every city has tennis courts, allowing half a million people play competitively.

 

D. 

The Dutch were the first Europeans to see Australia, in the early seventeenth century, but they found no opportunities to settle or trade. English explorer James Cook landed near Sydney 160 years later and claimed it for Britain. But still no one could think what to do with it. The American Revolution provided a solution. America had been used Australia as a dumping ground for criminals, mostly the unfortunate poor from cities created during the industrial revolution.

E. 

Australians used to live on a diet of lamb and potatoes. But with such a variety of climates and with so many ethic groups, the Australian diet has changed dramatically. Cities like Melbourne and Sydney are proud of their huge range of ethnic restaurants and multicultural cuisine, ranging from Italian to Tibetan. The barbecue is a national institution. It is an informal event for people and there is nothing more refreshing than a swim before eating fruit, fresh salads, and steaks.

ВАРИАНТ № 12  (9 КЛАСС - 2016)

Прочитайте тексты и установите соответствие между текстами АE и заголовками 16. Запишите свои ответы в таблицу. Используйте каждую цифру только один раз. В задании есть один лишний заголовок.

 

1. 

Food and medicine

2. 

A berry of beauty

3. 

Name meaning

   

4. 

Strawberry events

5. 

Strawberry recipe

6. 

Strawberry brief facts

A. 

A strawberry is a small fleshy red fruit used in desserts or eaten on its own. It is usually sold in small baskets known as punnets. Strawberries contain no fat and are very low on calories. They are a rich source of vitamin B6, vitamin C, and others. The United States is the top strawberry producing country in the world, followed by Spain. The typical modern strawberry is a hybrid from both North and South America.

B. 

Strawberries taste great with just about anything and they make a great snack on their own. The berry was known from the late middle ages but was not very popular then. People say that the name of the fruit comes from the way it was sold. Popular legend is that long time ago in London strawberries were very cheap. Children from London and other towns picked the berry themselves and then tied onto pieces of straw to sell in the town markets yelling “straws of berries” to get attention.

C. 

There are many strawberry festivals held around the world in celebration of strawberry season. Each strawberry festival is unique, but some of the common elements are often a youth show, carnival rides, a sell of homemade goods, musical performances, art contests, and baking competitions. You'll probably have the chance to buy fresh strawberries or strawberry pies. Most strawberry festivals charge a small entrance fee, but it is well worth it to experience strawberry related fun for an entire weekend.

 

D. 

North American native people called strawberries as berry of the heart and ate strawberries as a favored dessert. White settlers who came into contact with native people liked the berry. Then one doctor prescribed a diet of only strawberries believing that rheumatic illness could be cured if strawberries were eaten every day. According to his observations the leaves of the strawberry could be used in tea and would help to lose weight as well.

E. 

Whether you eat strawberries as juice, jam, or mix it with a cake, it has the same delicious taste. It is not just delicious, but also good for our health and appearance. The berries are widely used in cosmetics. During the late 20th century strawberry flavored beauty products have become very popular. There appeared many products such as facial cleansers, shampoos, soaps, and others.

ВАРИАНТ № 12  (9 КЛАСС - 2016)

Прочитайте тексты и установите соответствие между текстами и их заголовками: к каждому тексту, обозначенному буквами АG, подберите соответствующий заголовок, обозначенный цифрами. Используйте каждую цифру только один раз. В задании естьодин лишний заголовок.

 

1. 

Local legends

2. 

Special in many ways

3. 

Tourist аttraction

4. 

Diverse wildlife

   

5. 

Protection of the ecosystem

6. 

Extinct species

7. 

Scientific expeditions

8. 

Harsh climate

A. 

The world’s deepest lake, the Baikal, is in Siberia. It is also the largest freshwater lake in Eurasia and the oldest lake on the Earth. At least 1,500 unique species live there - they cannot be found anywhere else on the planet!  The water in the lake is so clear that if you drop a coin, it can be clearly seen a hundred feet below the water.

 

 

B. 

People who live in the Baikal region believe that it’s a unique and mysterious place. According to them, the water from Lake Baikal can cure different illnesses, gives you strength and clears your mind. They also say that the lake was formed millions of years ago when a huge, hot rock fell to earth. It melted the ice around and that was how Lake Baikal appeared. The story about the meteorite has, however, never been proved by scientists.

 

 

C. 

The lake has also become famous for its unique fish and birds that are not found in other waters. The lake is home to more than 1000 animal species. Among them there are the world’s only freshwater seals. There’s no evidence of how the seals got to the lake but they obviously enjoy their life there. Huge brown bears often come to the lake out of the forest to hunt and fish.

 

 

D. 

Though Lake Baikal is located in a very remote place, and is difficult to reach in autumn and winter, it attracts thousands of visitors every year. A chance to see this unique place is worth the long journey! On the banks of Lake Baikal you can stay in a modern, comfortable hotel, take part in hiking tours and enjoy the untouched natural beauty. People who have visited Lake Baikal once want to return to the place again and again.

 

 

E. 

However, the growing popularity of the lake and the industrial development of the region have caused ecological problems. The safety of this unique natural ecosystem has been discussed at an international level. Now Lake Baikal is on the list of heritage sites protected by UNESCO. A federal state law about the conservation of the lake was also supported in Russia.

 

 

F. 

The lake attracts not only tourists but also many wildlife researchers, biologists and even archaeologists.  The world famous explorer and scientist, Jacques Cousteau, and his team spent lots of time studying the deep waters of Lake Baikal. They also shot a film about their research that was broadcast by major TV channels all over the world.

 

 

G. 

Everyone considers the Siberian climate very severe, which is perfectly true for the Baikal region. The winters there are really freezing - the average temperature is as low as -25 degrees Celsius. Due to its location in the middle of the continent, the place is characterised by a sharp contrast between winter and summer temperatures. The summers are generally cool, with a few hot days. The sun shines brightly above the lake till late autumn.

ВАРИАНТ № 13  (9 КЛАСС - 2016)

Прочитайте тексты и установите соответствие между текстами и их заголовками: к каждому тексту, обозначенному буквами АG, подберите соответствующий заголовок, обозначенный цифрами. Используйте каждую цифру только один раз. В задании естьодин лишний заголовок.

 

1. 

A widely used aroma

 

2. 

Makes meals different

 

3. 

A relaxing effect

 

4. 

Scents and colours

 

   

5. 

Another unique characteristic

 

6. 

Holiday scents

 

7. 

Creating memories

 

8. 

Smelling emotions

A. 

Nothing brings back memories like a particular smell. Whether it's of Christmas pine, your grandma's fresh-baked gingerbread, or cookies, the scents of Christmas are truly special. In the days leading up to this day, the house fills with the wonderful rich, spicy smells of vanilla, cinnamon and ginger coming from the kitchen, promising delights to come.

 

 

B. 

Some of the most pleasant scents after a hard day are vanilla, lavender, and scents with cinnamon or ginger. Each combination of aromas can influence you positively. For example, vanilla's sweet scent can help you if you feel sad, lonely, or depressed. It is a naturally warming aroma. Cinnamon is good in case you feel tired; it also has a wonderful effect on your nerves, calming you down.

 

 

C. 

The ability to smell is linked to our ability to remember things. When you first smell a new thing, you connect it to an event, a person, or even a moment. As a result, later the smell of cookies might remind you of spending time at your grandmother's house when you were a small child. When you come across the smell a second or third time, the link is already there, ready to bring out a certain mood.

 

 

D. 

There are certain smells we can identify from a mile away  almost as if they're preprogrammed into our minds. One of them is vanilla. Today, vanilla is in our coffee, perfumes, tea, home products, body lotion, and everywhere! Both the scent and taste of vanilla are very strong and long-lasting. It is considered one of the most popular scents and flavours in the world.

 

 

E. 

A new study suggests that we can smell not only aromas but feelings as well  fear, happiness, disgust and joy. The experiments proved that we can find out how a person feels even if we neither see nor hear him. Nerves inside our nose take informative messages about the person and his emotional state to the brain. However, the mechanism of how this happens is not yet clear.

 

 

F. 

Our sense of smell does 80% of the job when we taste various foods. Without a sense of smell you can’t taste the difference between an apple and a potato or a glass of juice and a cup of cold coffee. This is why, when our nose is blocked by a cold, most foods seem tasteless. Our sense of smell becomes stronger when we are hungry.

 

 

G. 

There are many good reasons to believe that we all have our own particular smell. Research has proved that our smell might distinguish us from others just as our face does. Our smell is as personal as our fingerprints. For centuries the police have used this phenomenon to catch criminals. Maybe one day they will use our scent too.

 

 

ВА

Прочитайте тексты и установите соответствие между текстами и их заголовками: к каждому тексту, обозначенному буквами АG, подберите соответствующий заголовок, обозначенный цифрами. Используйте каждую цифру только один раз. В задании естьодин лишний заголовок.

 

1. 

Popular route

2. 

The secret of the old rocks

3. 

City of contrasts

4. 

Escape the heat

   

5. 

Common roots

6. 

Wildlife holidays

7. 

Holy waters

8. 

Movie magic

A. 

The world’s first university was established in India in 700 BC, where more than 10,500 students from around the world studied more than 60 different subjects. The training was conducted in Sanskrit. Nowadays Sanskrit is considered to be the mother of all the European languages. Its elements can be traced in most of them.

 

 

B. 

The most frequently visited place in India is Delhi, the capital of the country. It seems to be divided into two parts. New Delhi is a modern city with hi-tech architecture, tourist attractions and modern facilities. Old Delhi is several centuries old, with narrow streets, ancient churches and noisy bazaars. Lots of people live in slums without the most essential facilities like toilets and running water.

 

 

C. 

People all over the world know of India’s greatest river, the Ganges. It is the subject of thousands of prayers. The river looks extremely beautiful in the morning light. It is believed that bathing in the Ganges helps to cure all kinds of diseases and can improve your life in general. It has become a common ritual for pilgrims to gather by the river in the mornings.

 

 

D. 

The most famous and often visited Indian historic area is the Golden Triangle. It consists of the cities of Delhi, Agra and Jaipur. Lying in the south-east, Agra attracts tourists with its iconic image of the Taj Mahal palace, which is one of the eight wonders of the modern world. To the south-west is the colourful ‘Pink City’ of Jaipur, known for its Palace of Winds.

 

 

E. 

Indian climate differs from one part of the country to another. The air is cool and fresh in the mountains but in the plains it is usually very hot and dry so even a short trip can be very hard for an inexperienced traveller. That is why tourists are always glad to have a rest at one of the numerous stations in the hills. There they can enjoy a nice cup of real Indian tea, the most refreshing drink in such a climate.

 

 

F. 

India is not all history. It is also famous for its Bollywood industry, which produces love stories and action films. Indian films have always had their own style, rich in music and dancing. Though the industry is still developing, many of Indian film stars have received Oscar prizes and are known throughout the world. The home of Bollywood is Bombay, also known as a busy port and the country’s commercial centre.

 

 

G. 

There is an unusual treasure hidden in the Indian mountains. These are striking ancient temples cut in the rocks. For example, the Buddhist cave temples at Ajanta were built at least 2,000 years ago. Inside the caves on the walls tourists can see pictures of people’s lives and animal images. Some researchers believe that the interpretation of the images can help us to learn a lot about people’s lives in ancient times.

 

 

ВАРИАНТ № 14  (9 КЛАСС - 2016)

Прочитайте тексты и установите соответствие между текстами и их заголовками: к каждому тексту, обозначенному буквами АG, подберите соответствующий заголовок, обозначенный цифрами. Используйте каждую цифру только один раз. В задании естьодин лишний заголовок.

 

1. 

A talisman for luck

 

2. 

The wildlife of the country

3. 

The modern side of the country

 

4. 

Useful in many ways

 

   

5. 

A country profile

 

6. 

A false stereotype

 

7. 

Opportunities for different sports

 

8. 

The national sporting event

 

A. 

Thanks to television and films, Scotland is often seen by the rest of the world as a magical country, a land of misty mountains and the home of a lake creature called Nessie. This is the image most people have in mind. The truth, however, is completely different from the way the world thinks of Scotland. You may be surprised to find out that most Scots live in the lowlands and don’t believe in Nessie.

 

 

B. 

Scotland is much more than green hills and picturesque valleys. It’s dynamic and cosmopolitan, with a colourful history of invention and innovation. It’s the home of big businesses and a centre of new medical and scientific development. Each year, it hosts the world's biggest contemporary arts festivals. Scotland is an exciting place to live in, particularly for a young person.

 

 

C. 

Scottish heather, a small bush with flowers, is the floral symbol of the country. The colour of the flowers usually varies from purple to lilac, but they can also be white. In the past, the Scots believed that they would never be captured by enemies and would win victories if they wore white heather. Today, when getting married, girls add white heather to bouquets to bring fortune to their family.

 

 

D. 

Heather has grown in Scotland as far back as its history goes. This has turned it into a kind of household good. Heather honey is rich in minerals, and was traditionally used in medicinal drinks. Thousands of years ago, dried heather was used to make perfumes, and the tough stems were used to make ropes. On many of the northern Scottish islands, heather played a major role in the construction of houses.

 

E. 

Scotland is an excellent destination for open-air activities all year round. Its lakes and coastline are ideal for boating and rowing and have some of the best sea-kayaking in the world. The rivers are perfect for fishing and the mountains are wonderful for hill climbing and skiing. Scotland is also good for walkers. You can enjoy easy family walks or more difficult mountain hikes.

 

F. 

The Highland Games in Scotland date back to long before Scotland’s written history. The various athletics allowed men to demonstrate their physical power and the skills needed for battle and survival. The Games today still include the athletic competitions in their original form together with dancing contests. All the competitors wear kilts. Traditional bagpipe music accompanies all the events.

 

 

G. 

Scotland is a country that is part of the United Kingdom, occupying the northern third of the island of Great Britain. Edinburgh, the country's capital and second largest city, is one of Europe's financial centres. The population of the country is a little over 5 million people. The currency is the pound sterling although the Bank of Scotland issues its own notes and coins. 

ВАРИАНТ № 15  (9 КЛАСС - 2016)

Прочитайте тексты и установите соответствие между текстами и их заголовками: к каждому тексту, обозначенному буквами АG, подберите соответствующий заголовок, обозначенный цифрами. Используйте каждую цифру только один раз. В задании естьодин лишний заголовок.

 

1. 

Coffee in the New World

 

2. 

The original taste of coffee

 

3. 

A chance discovery

 

4. 

A place for communication

 

   

5. 

An illegal drink

 

6. 

A long journey to Europe

 

7. 

Growing coffee plants

 

8. 

Different recipes

 

A. 

Coffee was first found in Eastern Africa in the area we know today as Ethiopia. A popular legend says that one day a man called Kaldi was watching his goats eating grass and leaves. He noticed that after eating berries from a certain bush the goats became very active. Curious, Kaldi tried the berries himself. He found that these berries gave him additional energy. The story quickly spread throughout the region.

 

 

B. 

The first coffee berries were transported from Ethiopia to Arabian countries and then they were cultivated in Yemen. From there, coffee travelled to Turkey where coffee beans were roasted for the first time over open fires. The roasted beans were crushed and then boiled in water, creating the drink we enjoy today. Finally, in the seventeenth century coffee arrived in France, Britain and Italy.

 

 

C. 

The first coffeehouses opened in Mecca and quickly became popular centres of social and political activity. The government believed that coffee stimulated radical thinking, and the governor even thought it might unite his opposition. As a result, coffee was forbidden inMecca. The same thing happened in Turkey a century later, when coffee drinkers were severely punished.

 

 

D. 

They say, in 1776 the thirteen American colonies adopted coffee as their national drink instead of tea. When the British tried to raise the tax on tea, Americans refused to buy it and switched to coffee. In fact, by drinking coffee Americans demonstrated their political views and their dislike for the British  the famous tea-lovers.

 

E. 

The Italians drink their espresso with sugar, the Germans and Swiss add an equal part of hot chocolate, the Mexicans prefer it with cinnamon, and the Belgians  with chocolate. Coffee drinkers in the Middle East usually add cardamom and spices. The Egyptians, on the other hand, are extremely fond of pure, strong coffee. They add neither milk nor cream.

 

 

F. 

Plain and simple, nothing else. People discovered that if they drank coffee slowly they could taste bitterness, sourness, sweetness and saltiness. For a long time such a taste was new and more than enough. It needed no improvement. Only later did they start experimenting with coffee by adding milk, chocolate, and other flavors. However, nothing can hide the taste of real coffee.

 

 

G. 

When the first European coffeehouses opened, they quickly grew into places where people played chess, exchanged gossip, sang and danced. Coffeehouses were very popular because anyone could go and enjoy a cup of the drink. The price of a cup of coffee was reasonable. European coffee houses became meeting points for intellectuals, writers and businessmen.

ВАРИАНТ № 16  (9 КЛАСС - 2016)

Прочитайте тексты и установите соответствие между текстами и их заголовками: к каждому тексту, обозначенному буквами АG, подберите соответствующий заголовок, обозначенный цифрами. Используйте каждую цифру только один раз. В задании естьодин лишний заголовок.

 

1. 

Who supports the runners?

2. 

How it started

3. 

How to be sure of the winners

4. 

What to wear for a marathon

   

5. 

What it is like to run a marathon

6. 

How to get ready for a marathon

7. 

Who runs just for fun?

 

8. 

Can the landscape help?

A. 

Many cities in the United States hold marathons. The city of Chicago, Illinois, has its marathon once a year. The running area inChicago is almost absolutely flat. The scenery is amazingly beautiful. This fact has helped some runners set some of the world’s fastest times there. Many of them hope to go back again.

 

 

B. 

Some people know that they have no chance of winning but they participate anyway. They are not officially registered for the race - they just start running with the crowd. These runners are called “bandits.” Many of them finish the race hours after the serious runners but they are happy to cross the finish line anyway.

 

 

C. 

An important battle took place in Marathon in Greece more than two thousand years ago. Greece’s army won the battle. A Greek runner was sent to Athens to announce the victory. The man ran for about 40 km at top speed. He announced his message. Then he fell to the ground and died. In memory of the great runner a marathon was included in the Olympic Games.

 

D. 

Recently, a British writer and journalist, Mario Roter, ran in a marathon. He thought it would be fun to write an article about what people felt during the race. Later he said that running forty-two kilometers was no fun. Many people agree that there is no pleasure in running a marathon, but afterwards it feels great to know that you have done it.

 

 

E. 

People usually start training for a marathon in early spring. They run often and gradually increase the length of their runs from eight to twelve and even to twenty kilometers. At this point it is necessary to build up the needed strength. In the months before the race, the average participant runs a total of more than eight hundred kilometers.

 

 

F. 

Usually there are lots of runners. The line of people stretches back hundreds of meters. As the winners are awarded prize money it is important to avoid cheating. In order to track all the participants, special chips are used. When the participants register for the marathon, they get their identification number and computer chips.  The chips are activated at the start of the race and keep time.

 

 

G. 

A marathon is usually a big public event. In the city where the marathon is held hundreds of people give their time and effort to make the race possible. The city police block traffic on the main roads. Some runners thank the officers as they run by. Every few kilometers there are volunteers who offer water to the runners. Lots of people cheer.

 

ВАРИАНТ № 17  (9 КЛАСС - 2016)

Прочитайте тексты и установите соответствие между текстами и их заголовками: к каждому тексту, обозначенному буквами АG, подберите соответствующий заголовок, обозначенный цифрами. Используйте каждую цифру только один раз. В задании естьодин лишний заголовок.

 

1. 

Our never sleeping part

 

2. 

Influence on academic results

3. 

Advantages of bedtime routine

4. 

Good sleeping conditions

 

   

5. 

Pre-sleep activities

6. 

Sleeping classes

 

7. 

A negative effect on health

8. 

The mechanism of dreaming

A. 

Recent research shows that lots of teenagers are suffering from lack of sleep which is causing problems with their behaviour and learning. As a result schools have introduced a new lesson where students are given more of an idea of the importance of sleep. According to medical data, they should get at least nine hours' sleep a night. Young people should realise that sleep is very important if they want to do well at school and enjoy life.

 

 

B. 

It’s common knowledge that people sleep best in a dark room that is slightly cool. It's important to close your blinds or curtains and make sure they're heavy enough to block out the light. You can add extra blankets or wear pyjamas if you're cold. Note that an uncomfortable bed, heat and noise can ruin a good night's sleep. A TV is also a distraction.

 

 

C. 

If you don’t sleep enough or have a disturbed sleep you can get heart disease, high blood pressure and even diabetes. People who don't sleep enough get colds and flu more often than those who do. Moreover, recent research proves that poor sleepers gain weight more easily than people who sleep well. Extra kilos are the reaction of the body to the stressful sleepless situation.

 

 

D. 

Think twice before watching a movie late at night. Don’t spend sleeping time on endless computer games. The thing is that by getting up and going to bed at the same time every day, even on weekends, you teach your body to follow a healthy regime. You'll quickly fall asleep, will have a good night’s sleep and will wake up at the expected time feeling fresh and active.

 

 

E. 

Some easy steps can prepare your body for going to sleep. You probably have some regular bedtime habits, even if you don’t realise it. Brushing your teeth, taking a shower, reading a book and setting your alarm clock may all be part of your evening routine. Remember that you should perform these activities in the same manner and in the same order every night.

 

 

F. 

Sleepless nights at exam time are very bad for students. If you prepare for your exam late at night, it will not improve your performance during the exam. On the contrary, it will affect your ability to concentrate, to react quickly and to analyse information. Recent research shows that the results of the tests written after sleepless nights are lower than usual grades.

 

 

G. 

If you think that during sleep, the brain rests, you're mistaken. It's a common myth about sleep. Sleep is definitely needed  but it's not your brain that needs to rest, it's your body that needs to relax. Your brain is still working when you're asleep, controlling physical functions like breathing and your heart beat. However, because the brain does not have to go through the constant stress of thinking, you get up refreshed and recharged.

 

ВАРИАНТ № 18  (9 КЛАСС - 2016)

Прочитайте тексты и установите соответствие между текстами и их заголовками: к каждому тексту, обозначенному буквами АG, подберите соответствующий заголовок, обозначенный цифрами. Используйте каждую цифру только один раз. В задании естьодин лишний заголовок.

 

1. 

Good for everyone

 

2. 

Easy to care for

 

3. 

Part of culture

 

4. 

Screen fashion

 

   

5. 

An innovative idea

 

6. 

Working clothes

 

7. 

Jeans’ labels

 

8. 

Clothes to protest

A. 

Jeans are one of fashion's most long enduring trends. Cowboys wear them but so do supermodels, farmers, presidents and housewives. Ask any group of people why they wear jeans and you will get a range of answers. For some they're comfortable and easy for others they're trendy and cool. Jeans mean different things to different people, but they are popular everywhere.

 

 

B. 

Americans do not have a national folk dress with a long tradition. Blue jeans are probably the most recognisable article of American clothing. They have been part of American life for over 125 years. Blue denim jeans became not only an expression of American fashion but also an element of American identity known around the world.

 

 

C. 

Jeans were first designed as trousers for farmers and miners in the states of the American West. They quickly grew popular with common people, including cowboys, factory employees and railroad builders. The new trousers were made from a very strong material which did not wear out easily. However, at the same time jeans were very practical and comfortable to wear.

 

 

D. 

Pockets were the weak point of the miners' clothes  they easily tore away from the jeans. A man called Jacob Davis had the idea of using metal rivets (fasteners) to hold the pockets and the jeans together so that they wouldn't tear. Davis wanted to patent his idea, but he didn't have enough money, so he offered Levi Strauss a deal if Strauss paid for the patent. Strauss accepted and started making jeans.

 

 

E. 

By the middle of the twentieth century, these heavy cotton trousers were a symbol of opposition for young artists and writers. College students started to wear them to show they were against the Vietnam War. The new trousers were banned in American schools from coast to coast and sometimes in theatres and cinemas.

 

 

F. 

Jeans are good because they don't show the dirt. You can easily go a month without washing them and they don't look shocking. They don't need to be washed as often as other trousers and you don't need to iron them. What's more, because of the strong material you can wear your favorite jeans for years. Even the occasional hole or spot doesn’t spoil them at all.

 

G. 

In the 30s and 40s many people began to spend their spare time watching movies where adventurous cowboys rode horses, fought bad guys and wore blue jeans. The actors made jeans popular in movies and everyone wanted to wear them. Young people wished to imitate the casual “cowboyish” look they saw in films, and they began to wear jeans as casual wear.

 

ВАРИАНТ № 19  (9 КЛАСС - 2016)

Прочитайте тексты и установите соответствие между текстами и их заголовками: к каждому тексту, обозначенному буквами АG, подберите соответствующий заголовок, обозначенный цифрами. Используйте каждую цифру только один раз. В задании естьодин лишний заголовок.

 

1. 

A false stereotype

2. 

An unlucky symbol

 

3. 

A colourful present

 

4. 

A world famous journey

 

   

5. 

One and the same name

 

6. 

The national food

 

7. 

A talisman for luck

8. 

Old holiday traditions

A. 

When you think of the Irish, you often picture a red-haired person dressed all in green. However, it's just a popular myth which has grown into a tradition, particularly in the United States. It's customary in Ireland to wear green clothes only on St. Patrick's Day. Lots of people are tricked by this cliché. However, originally, the colour associated with Saint Patrick was not green, but blue.

 

 

B. 

In Ireland, the colour green was long considered to bring bad fortune. The reason is that in Irish folklore green is the favourite color ofthe Good People (the proper name for fairies). Myths run that they are likely to steal people, especially children, who wear too much of the colour. In the past, a girl would never wear anything green on her wedding day.

 

 

C. 

Christmas is a very important celebration in Ireland. After dinner on Christmas Eve, it is common for families to leave milk and bread on the table as a sign of friendliness and kindness. Another custom is to leave the door unlocked. A lit candle is left in a window during the night. It represents help for any traveller who is passing by.

 

 

D. 

Potatoes form the basis for many traditional Irish dishes. They are eaten boiled, mashed, fried and baked. Potatoes are mixed with cabbage or green onions to make traditional Irish dishes. They are also made into potato cakes and used in soups or stews. It’s common to find potatoes cooked in two different ways on the same dinner plate.

 

 

E. 

The shamrock is still a popular sign of good fortune in Ireland. It is believed that anyone who possesses one will be blessed with fortune in everything, even in gambling, and will be saved from the evil of witches. There are certain conditions to be met so that its power remains effective: the owner of the shamrock must keep it away from the public eye and never give it to anyone else.

 

 

F. 

The name Gulliver is known to everyone due to the book or the film about the fantastic trip of the English doctor. However, not all of us remember the name of the author undefined  Jonathan Swift, an Irish writer, who was born in Dublin, Ireland, in 1667. In his book Gulliver’s Travels a man went to an island where strange tiny people lived. His life there was full of adventures and dangers, he met there friends and enemies.

 

G. 

Ireland is known as the native land of limericks  short humorous poems that have five lines. They make people laugh and are easy to remember! Lots of poets and writers were fond of limericks. The word ‘limerick’ probably comes from the Irish town of Limerick. The short poem has made the town known all over the world.

ВАРИАНТ № 20  (9 КЛАСС - 2016)

Прочитайте тексты и установите соответствие между текстами и их заголовками: к каждому тексту, обозначенному буквами АG, подберите соответствующий заголовок, обозначенный цифрами. Используйте каждую цифру только один раз. В задании естьодин лишний заголовок.

 

1. 

Street performers

2. 

Eating together

3. 

Important for the whole country

4. 

Lifting weights

   

5. 

Cooking competition

6. 

Cheese Rolling

7. 

In memory of the past events

8. 

A cookery sprint

A. 

A woolsack race is one of the British local festivals that could be called strange. It started in Tetbury, a wool town, in the 17th century when young men wanted to demonstrate their physical strength. Since then, every spring men and women compete in teams to carry heavy woolsacks up and down the hill. The race events are complemented by a funfair and musical entertainments.

 

 

B. 

Midsummer was the time for the Cheese Rolling Ceremony in many places. Competitors gathered at the top of a hill. The Master of the Ceremonies let a heavy head of cheese roll down the hill. Brave runners raced down to be the first to catch it. However, the event was cancelled in 2010 due to safety reasons.

 

 

C. 

Another cheese ceremony is popular in the village of Randwick. On the first Sunday in May people roll three cheeses from right to left around the church. After rolling, the villagers cut up and share one of the cheeses. They believe that eating cheese brings health to their families.

 

 

D. 

The village of Marshfield, England, is famous for its Paperboys procession. People dressed in paper costumes go through the streets. They start from the market place and perform the town’s unique character play along the road. By noon they have done more than six performances for several hundred people.

 

 

E. 

Every January Up Helly Aa is celebrated in Scotland. People dressed in Viking costumes and helmets go through the streets of Lerwick. They hold flaming torches, sticks with the special material on the top which burns in order to give light. The strongest participants carry a full size model of a Viking ship to an open field. There the people throw lit torches into the ship and burn it.

 

 

F. 

Melbourne Cup Day is held in Australia, in November. Although Cup Day is a public holiday only in the city of Melbourne, the rest of the country refuses to be left out of the event. People gather around televisions and computers, whether at work, at home, or wherever they are, just to watch this world famous horse race. This event is often called ‘the race that stops the nation’.

 

G. 

In a village in Eastern England, an unusual race takes place every year. Three groups take part in the race  adults, children (under 11s) and teenagers. Each participant receives a frying pan with a pancake and has to race from one end of a field to the other, throwing the pancake into the air and catching it in the frying pan without dropping it. The winner is the first to cross the line.

 

ВАРИАНТ № 21  (9 КЛАСС - 2016)

Прочитайте тексты и установите соответствие между текстами и их заголовками: к каждому тексту, обозначенному буквами АG, подберите соответствующий заголовок, обозначенный цифрами. Используйте каждую цифру только один раз. В задании естьодин лишний заголовок.

 

1. 

With care for the environment

2. 

A favourable location

3. 

Medicine from birds

4. 

Annual events

   

5. 

Names to remember

6. 

Getting around the town

7. 

A historical building

8. 

A green town

A. 

Cheltenham started as a small market town that later became one of the most fashionable health resorts in Britain. Once, in 1716 people noticed pigeons eating crystals in a meadow outside the town. These were crystals of salt which helped King George III and the members of the royal family treat their illnesses. Since then, pigeons have been a symbol of Cheltenham Spa.

 

 

B. 

Cheltenham is a small, ancient town in England situated to the north-west of Oxford. It lies below the highest point of the Cotswold Hillsand above the level of the River Severn. The town is also called ‘The Western Gateway’. Thanks to its position it has become a popular tourist attraction.

 

C. 

The Pittville Pump Room is the largest of the spa constructions in Cheltenham. It is a magnificent, old house of classical style made of white stone. The Pump Room was designed for balls and entertainments and, of course, for taking the medicinal waters. Today, it is still used as a concert hall at festival time and for weddings, and of course, visitors can still take the waters  if they so wish!

 

D. 

Tourism is highly developed in Cheltenham and the City Council has to reduce the negative impact of tourists on the town. A lot has been done. The town has developed walking, cycling and public transport routes as an alternative to cars. There is a pool of bicycles that tourists can use for short distance journeys. The local services try to recycle all the packaging, plastic bottles and batteries left by tourists.

 

 

E. 

Many famous people of the past have connections with Cheltenham. Gustav Holst, a well-known English composer of the 19th century, was born in the town. His house is now his birthplace museum. Visitors may learn a lot about Holst's life, his music and his family. The famous Antarctic explorer Edward Wilson was also born in Cheltenham. Now a statue of him stands in the Long Gardens.

 

F. 

The beauty and wonderful sights of Cheltenham continue to attract visitors from countries all over the world. For many years people have enjoyed the beauty of Cheltenham's parks, open spaces and the general greenery of the town. Cheltenham has been described as ‘a town within a park’. Montpellier place, which is famous for its wonderful flowers, lies in the centre of the town.

 

 

G. 

Thousands of visitors come to Cheltenham to take part in the festivals which are held in the town every year. The town organises literature, music, jazz and science festivals, attracting names with a national and international reputation from each field. Besides all this, for a week in March the town becomes the centre of the National Gold Cup in horse racing.

 

ВАРИАНТ № 22  (9 КЛАСС - 2016)

Прочитайте тексты и установите соответствие между текстами и их заголовками: к каждому тексту, обозначенному буквами АG, подберите соответствующий заголовок, обозначенный цифрами. Используйте каждую цифру только один раз. В задании естьодин лишний заголовок.

 

1. 

A widely used aroma

 

2. 

Makes meals different

 

3. 

A relaxing effect

 

4. 

Scents and colours

 

   

5. 

Another unique characteristic

 

6. 

Holiday scents

 

7. 

Creating memories

 

8. 

Smelling emotions

A. 

Nothing brings back memories like a particular smell. Whether it's of Christmas pine, your grandma's fresh-baked gingerbread, or cookies, the scents of Christmas are truly special. In the days leading up to this day, the house fills with the wonderful rich, spicy smells of vanilla, cinnamon and ginger coming from the kitchen, promising delights to come.

 

 

B. 

Some of the most pleasant scents after a hard day are vanilla, lavender, and scents with cinnamon or ginger. Each combination of aromas can influence you positively. For example, vanilla's sweet scent can help you if you feel sad, lonely, or depressed. It is a naturally warming aroma. Cinnamon is good in case you feel tired; it also has a wonderful effect on your nerves, calming you down.

 

 

C. 

The ability to smell is linked to our ability to remember things. When you first smell a new thing, you connect it to an event, a person, or even a moment. As a result, later the smell of cookies might remind you of spending time at your grandmother's house when you were a small child. When you come across the smell a second or third time, the link is already there, ready to bring out a certain mood.

 

 

D. 

There are certain smells we can identify from a mile away  almost as if they're preprogrammed into our minds. One of them is vanilla. Today, vanilla is in our coffee, perfumes, tea, home products, body lotion, and everywhere! Both the scent and taste of vanilla are very strong and long-lasting. It is considered one of the most popular scents and flavours in the world.

 

 

E. 

A new study suggests that we can smell not only aromas but feelings as well  fear, happiness, disgust and joy. The experiments proved that we can find out how a person feels even if we neither see nor hear him. Nerves inside our nose take informative messages about the person and his emotional state to the brain. However, the mechanism of how this happens is not yet clear.

 

 

F. 

Our sense of smell does 80% of the job when we taste various foods. Without a sense of smell you can’t taste the difference between an apple and a potato or a glass of juice and a cup of cold coffee. This is why, when our nose is blocked by a cold, most foods seem tasteless. Our sense of smell becomes stronger when we are hungry.

 

 

G. 

There are many good reasons to believe that we all have our own particular smell. Research has proved that our smell might distinguish us from others just as our face does. Our smell is as personal as our fingerprints. For centuries the police have used this phenomenon to catch criminals. Maybe one day they will use our scent too.

ВАРИАНТ № 23  (9 КЛАСС - 2016)

Прочитайте тексты и установите соответствие между текстами и их заголовками: к каждому тексту, обозначенному буквами АG, подберите соответствующий заголовок, обозначенный цифрами. Используйте каждую цифру только один раз. В задании естьодин лишний заголовок.

 

1. 

A convenient food

 

2. 

Good for health

 

3. 

Part of culture

 

4. 

Unclear origins

 

   

5. 

Pasta ingredients

6. 

A useful invention

 

7. 

Classless food

 

8. 

An expensive food

A. 

You can enjoy pasta on any budget, and you can have a different and delicious low-cost meal every day of the week. Just serve it with a different sauce. It’s easy to make a healthy meal in minutes that will satisfy your whole family. Pasta tastes good and it's healthy. It also has a long shelf life, so you can keep it in the kitchen cupboard until you need to cook a meal.

 

 

B. 

It's rather difficult to find out where pasta comes from. Archeologists believe that cooking noodles was already quite a common practice in Arabic countries in the 5th century AD. At the same time, the Chinese were also making a noodle-like food. The way it reached Europe is still a mystery, though there are many theories  some believe that travelling Arabs from Central Asia were responsible for bringing early forms of pasta to the West.

 

 

C. 

Despite its reputation, pasta is a low-calorie dish which is good for you. With only 200 calories per cup and a gram of fat, it is perfect even for those on a strict diet. Pasta is naturally packed with vitamins and necessary minerals so it is heart-healthy as well. Pasta may help people to control their blood sugar and weight so it is recommended as part of a well-balanced diet.

 

 

D. 

It seems that pasta was first introduced to Italy in the eighth century. Its ‘home’ was the southern island of Sicily. Prior to its wide use in the 13th century, pasta was considered to be a luxury product in Italy. The reason was that it took a lot of time to make it as all the work was done by hand. By 1400, pasta was sold in shops, but at night a guard was kept to protect the high-priced goods.

 

 

E. 

Only in the 16th century did pasta become popular and affordable to all classes of people. In the city of Naples a machine was made that allowed huge quantities of pasta to be produced. It turned pasta into a cheap food. The number of pasta shops in Naples grew to three hundred. From Naples pasta began to conquer the rest of the country.

 

F. 

By the 17th century, machine-made pasta had become part of the daily meal among both rich and poor. Pasta was sold as street food by people called maccaronaros, who cooked it over an open fire. It was eaten on the spot with bare hands, with no sauce. The wealthy, who did not eat with their hands, preferred fresh pasta with cheeses and meat. With the introduction of the fork, pasta began to be served at special occasions all over Italy.

 

 

G. 

When most people talk about pasta, they usually think of Italy and its cooking traditions. The country and its dish have become synonymous. When Italians immigrated to different countries all over the world, they took their pasta with them to each area that they inhabited along with other customs and habits and thus pasta has become the world's favourite food.

 

ВАРИАНТ № 24  (9 КЛАСС - 2016)

Прочитайте тексты и установите соответствие между текстами и их заголовками: к каждому тексту, обозначенному буквами АG, подберите соответствующий заголовок, обозначенный цифрами. Используйте каждую цифру только один раз. В задании естьодин лишний заголовок.

 

1. 

Shop and travel

2. 

They don’t want to change anything

3. 

A shop for the rich

4. 

Shopping on water

   

5. 

Children's dreamland

6. 

Shopping alone

7. 

A taste from the past

8. 

Not new but good and cheap

A. 

Hamleys is the best toyshop in the UK. It has seven floors of toys, games and sweets. The shop assistants often dress up in costumes of famous characters from fairy tales and stories. Because of this the place looks like a magic land. The atmosphere is wonderful, and the shop really has everything young ones could ever want.

 

B. 

Harrods, is perhaps London's most famous department store, named after Charles Harrods, who opened a family grocer's shop on the site in 1849. The little shop has grown and become a symbol for expensive and glamorous shopping. It contains over 300 departments selling luxury items, from furniture and ladies fashion to sports equipment. Harrods continues to astonish customers from around the world with first-class service and product quality.

 

C. 

The Old Umbrella Shop is one of the shops that hasn’t changed from the early twentieth century. Inside it looks exactly as it was years ago. There are old telephones, shelves, shop windows and an umbrella museum. What's more, it still sells umbrellas and doesn’t want to specialize in anything else. The shop has faithful clients who have been coming there for years.

 

D. 

Muara Kuin is an unusual river market in Indonesia. With the sunrise people from local villages arrive here by boat with fresh fruit and vegetables. Buyers get to the market in the same way. Nobody goes on shore, all business is done from boats. If you wish to get a cup of tea, there are special motor boats selling drinks and cookies.

 

E. 

The city of Dongguan in China has the world's largest shopping mall. There are seven zones in it, which are designed to look like world's cities and regions. In no time you'll get from Paris to Amsterdam. Take a lift and it will take you from Europe to Canada. There's even a small river with boats, where you can relax and take a trip after shopping.

 

F. 

Hope and Greenwood is a sweet shop in London. It offers traditional British sweets. They were popular in the 50s and 70s. Now you can’t find them anywhere else, because chocolate factories use new modern recipes and ingredients. This shop offers a unique opportunity to try the desserts of the previous century.

 

G. 

Oxfam is a chain of shops all over the world, which sells very cheap things. Its aim is to attract shoppers by offering them many second-hand and donated things of good quality. Also anyone can come and bring the books or clothes they no longer need. Oxfam takes everything: from stamps to furniture.


Предварительный просмотр:


Подписи к слайдам:

Слайд 1

ОГЭ Устная Часть - Задание 3 - Новые монологи! http://oge.fipi.ru/os/xmodules/qprint/index.php?theme_guid=6AE467806D39AABB4E933C01D8EB127E&proj_guid=8BBD5C99F37898B6402964AB11955663

Слайд 2

Task 3. You are going to give a talk about your school holidays. You will have to start in 1.5 minutes and speak for not more than 2 minutes (10–12 sentences). Remember to say: when you have school holidays; what school holidays you would make longer, and why; what you enjoy doing during your school holidays; what your attitude to school holidays is. You have to talk continuously. Preparation

Слайд 3

Task 3. You are going to give a talk about your school holidays. You will have to start in 1.5 minutes and speak for not more than 2 minutes (10–12 sentences). Remember to say: when you have school holidays; what school holidays you would make longer, and why; what you enjoy doing during your school holidays; what your attitude to school holidays is. You have to talk continuously. Recording

Слайд 4

Task 3. You are going to give a talk about travelling. You will have to start in 1.5 minutes and speak for not more than 2 minutes (10–12 sentences). Remember to say: why people like travelling; what means of transport is the best for travelling, and why; what places in Russia you would like to visit; what your attitude to travelling is. You have to talk continuously. Preparation

Слайд 5

Task 3. You are going to give a talk about travelling. You will have to start in 1.5 minutes and speak for not more than 2 minutes (10–12 sentences). Remember to say: why people like travelling; what means of transport is the best for travelling, and why; what places in Russia you would like to visit; what your attitude to travelling is. You have to talk continuously. Recording

Слайд 6

Task 3. You are going to give a talk about reading books. You will have to start in 1.5 minutes and speak for not more than 2 minutes (10–12 sentences). Remember to say: whether reading is popular with teenagers, and why, or why not; what kind of books you like reading; why many people prefer e-books to paper books; what your attitude to reading is. You have to talk continuously. Preparation

Слайд 7

Task 3. You are going to give a talk about reading books. You will have to start in 1.5 minutes and speak for not more than 2 minutes (10–12 sentences). Remember to say: whether reading is popular with teenagers, and why, or why not; what kind of books you like reading; why many people prefer e-books to paper books; what your attitude to reading is. You have to talk continuously. Recording

Слайд 8

Task 3. You are going to give a talk about travelling. You will have to start in 1.5 minutes and speak for not more than 2 minutes (10–12 sentences). Remember to say: why most people enjoy travelling; what people like doing while travelling; what place you would like to go to, and why; what your attitude to travelling is. You have to talk continuously. Preparation

Слайд 9

Task 3. You are going to give a talk about travelling. You will have to start in 1.5 minutes and speak for not more than 2 minutes (10–12 sentences). Remember to say: why most people enjoy travelling; what people like doing while travelling; what place you would like to go to, and why; what your attitude to travelling is. You have to talk continuously. Recording

Слайд 10

Task 3. You are going to give a talk about keeping fit. You will have to start in 1.5 minutes and speak for not more than 2 minutes (10–12 sentences). Remember to say: why doing sport is very important for modern teenagers; what else besides sport young people do to keep fit; what you enjoy doing in your free time; what your attitude to doing sports activities is. You have to talk continuously. Preparation

Слайд 11

Task 3. You are going to give a talk about keeping fit. You will have to start in 1.5 minutes and speak for not more than 2 minutes (10–12 sentences). Remember to say: why doing sport is very important for modern teenagers; what else besides sport young people do to keep fit; what you enjoy doing in your free time; what your attitude to doing sports activities is. You have to talk continuously. Recording

Слайд 12

Task 3. You are going to give a talk about animals. You will have to start in 1.5 minutes and speak for not more than 2 minutes (10–12 sentences). Remember to say: what wild animals live in your region; whether it is a good idea to keep a wild animal as a pet, and why; why people build zoos in cities and towns; what your attitude to zoos is. You have to talk continuously. Preparation

Слайд 13

Task 3. You are going to give a talk about animals. You will have to start in 1.5 minutes and speak for not more than 2 minutes (10–12 sentences). Remember to say: what wild animals live in your region; whether it is a good idea to keep a wild animal as a pet, and why; why people build zoos in cities and towns; what your attitude to zoos is. You have to talk continuously. Recording

Слайд 14

Task 3. You are going to give a talk about your school. You will have to start in 1.5 minutes and speak for not more than 2 minutes (10–12 sentences). Remember to say: what your typical school day is like; what subjects at school you find most useful for your future, and why; what you are going to do when you leave school; what your attitude to school life is. You have to talk continuously. Preparation

Слайд 15

Task 3. You are going to give a talk about your school. You will have to start in 1.5 minutes and speak for not more than 2 minutes (10–12 sentences). Remember to say: what your typical school day is like; what subjects at school you find most useful for your future, and why; what you are going to do when you leave school; what your attitude to school life is. You have to talk continuously. Recording

Слайд 16

Task 3. You are going to give a talk about school life. You will have to start in 1.5 minutes and speak for not more than 2 minutes (10–12 sentences). Remember to say: what your weekday is like; what you like about your school most of all; whether you prefer classroom learning or online learning, and why; what your attitude to your school life is. You have to talk continuously. Preparation

Слайд 17

Task 3. You are going to give a talk about school life. You will have to start in 1.5 minutes and speak for not more than 2 minutes (10–12 sentences). Remember to say: what your weekday is like; what you like about your school most of all; whether you prefer classroom learning or online learning, and why; what your attitude to your school life is. You have to talk continuously. Recording

Слайд 18

Task 3. You are going to give a talk about sports. You will have to start in 1.5 minutes and speak for not more than 2 minutes (10–12 sentences). Remember to say: why a lot of young people do sports nowadays; what sports clubs and teams there are in your school; what you do to keep fit; what your attitude to doing sports activities is. You have to talk continuously. Preparation

Слайд 19

Task 3. You are going to give a talk about sports. You will have to start in 1.5 minutes and speak for not more than 2 minutes (10–12 sentences). Remember to say: why a lot of young people do sports nowadays; what sports clubs and teams there are in your school; what you do to keep fit; what your attitude to doing sports activities is. You have to talk continuously. Recording

Слайд 20

Task 3. You are going to give a talk about environmental problems. You will have to start in 1.5 minutes and speak for not more than 2 minutes (10–12 sentences). Remember to say: why people worry about environmental problems nowadays; what the most serious environmental problem in the place where you live is; what young people can do to improve the ecological situation; what your attitude to environmental problems is. You have to talk continuously. Preparation

Слайд 21

Task 3. You are going to give a talk about environmental problems. You will have to start in 1.5 minutes and speak for not more than 2 minutes (10–12 sentences). Remember to say: why people worry about environmental problems nowadays; what the most serious environmental problem in the place where you live is; what young people can do to improve the ecological situation; what your attitude to environmental problems is. You have to talk continuously. Recording

Слайд 22

Task 3. You are going to give a talk about your school. You will have to start in 1.5 minutes and speak for not more than 2 minutes (10–12 sentences). Remember to say: what you like about your school most of all; how many lessons a day you usually have; what school subjects you have chosen for your exams, and why; what your attitude to the number of subjects you have to learn is. You have to talk continuously. Preparation

Слайд 23

Task 3. You are going to give a talk about your school. You will have to start in 1.5 minutes and speak for not more than 2 minutes (10–12 sentences). Remember to say: what you like about your school most of all; how many lessons a day you usually have; what school subjects you have chosen for your exams, and why; what your attitude to the number of subjects you have to learn is. You have to talk continuously. Recording

Слайд 24

Task 3. You are going to give a talk about your free time. You will have to start in 1.5 minutes and speak for not more than 2 minutes (10–12 sentences). Remember to say: whether you have a lot of free time, and why, or why not; what you enjoy doing in your free time; what your Sunday afternoons are like; what your attitude to planning your free time is. You have to talk continuously. Preparation

Слайд 25

Task 3. You are going to give a talk about your free time. You will have to start in 1.5 minutes and speak for not more than 2 minutes (10–12 sentences). Remember to say: whether you have a lot of free time, and why, or why not; what you enjoy doing in your free time; what your Sunday afternoons are like; what your attitude to planning your free time is. You have to talk continuously. Recording

Слайд 26

Task 3. You are going to give a talk about films. You will have to start in 1.5 minutes and speak for not more than 2 minutes (10–12 sentences). Remember to say: what kinds of films modern teenagers enjoy; where you prefer watching films: on TV, on the Internet or in the cinema, and why; what film you have seen recently, what it was about; what your attitude to watching films as a way to spend your free time is You have to talk continuously. Preparation

Слайд 27

Task 3. You are going to give a talk about films. You will have to start in 1.5 minutes and speak for not more than 2 minutes (10–12 sentences). Remember to say: what kinds of films modern teenagers enjoy; where you prefer watching films: on TV, on the Internet or in the cinema, and why; what film you have seen recently, what it was about; what your attitude to watching films as a way to spend your free time is You have to talk continuously. Recording

Слайд 28

Task 3. You are going to give a talk about school homework. You will have to start in 1.5 minutes and speak for not more than 2 minutes (10–12 sentences). Remember to say: how long it takes you to do your homework; what subject you usually start with, and why; whether schoolchildren should be given more or less homework, and why; what your attitude to school homework is. You have to talk continuously. Preparation

Слайд 29

Task 3. You are going to give a talk about school homework. You will have to start in 1.5 minutes and speak for not more than 2 minutes (10–12 sentences). Remember to say: how long it takes you to do your homework; what subject you usually start with, and why; whether schoolchildren should be given more or less homework, and why; what your attitude to school homework is. You have to talk continuously. Recording

Слайд 30

Task 3. You are going to give a talk about photography. You will have to start in 1.5 minutes and speak for not more than 2 minutes (10–12 sentences). Remember to say: why people like taking photos; why taking photos is more popular today than it was in the past; what the best photo you have ever taken is; what your attitude to taking photos is. You have to talk continuously Preparation

Слайд 31

Task 3. You are going to give a talk about photography. You will have to start in 1.5 minutes and speak for not more than 2 minutes (10–12 sentences). Remember to say: why people like taking photos; why taking photos is more popular today than it was in the past; what the best photo you have ever taken is; what your attitude to taking photos is. You have to talk continuously Recording

Слайд 32

Task 3. You are going to give a talk about TV. You will have to start in 1.5 minutes and speak for not more than 2 minutes (10–12 sentences). Remember to say : why people spend time watching TV; what most teenagers prefer: watching TV or browsing the Internet, and why; whether there is a TV programme you really like; what your attitude to watching TV as a way to spend your free time is. You have to talk continuously. Preparation

Слайд 33

Task 3. You are going to give a talk about TV. You will have to start in 1.5 minutes and speak for not more than 2 minutes (10–12 sentences). Remember to say : why people spend time watching TV; what most teenagers prefer: watching TV or browsing the Internet, and why; whether there is a TV programme you really like; what your attitude to watching TV as a way to spend your free time is. You have to talk continuously. Recording

Слайд 34

Task 3. You are going to give a talk about TV. You will have to start in 1.5 minutes and speak for not more than 2 minutes (10–12 sentences). Remember to say: whether watching TV is a popular pastime with teenagers, and why, or why not; how many hours a week you watch TV; what TV programme is the most popular within your family; what your attitude to TV is. You have to talk continuously . Preparation

Слайд 35

Task 3. You are going to give a talk about TV. You will have to start in 1.5 minutes and speak for not more than 2 minutes (10–12 sentences). Remember to say: whether watching TV is a popular pastime with teenagers, and why, or why not; how many hours a week you watch TV; what TV programme is the most popular within your family; what your attitude to TV is. You have to talk continuously . Recording

Слайд 36

Task 3. You are going to give a talk about your best friend. You will have to start in 1.5 minutes and speak for not more than 2 minutes (10–12 sentences). Remember to say: what people need friends for; how long you and your friend have known each other; what you enjoy doing together; what your attitude to friendship is. You have to talk continuously. Preparation

Слайд 37

Task 3. You are going to give a talk about your best friend. You will have to start in 1.5 minutes and speak for not more than 2 minutes (10–12 sentences). Remember to say: what people need friends for; how long you and your friend have known each other; what you enjoy doing together; what your attitude to friendship is. You have to talk continuously. Recording

Слайд 38

Task 3. You are going to give a talk about your school. You will have to start in 1.5 minutes and speak for not more than 2 minutes (10–12 sentences). Remember to say: what your typical school day is like; what your favourite subject is, and why what you like most about your school; what your attitude to your school life is. You have to talk continuously. Preparation

Слайд 39

Task 3. You are going to give a talk about your school. You will have to start in 1.5 minutes and speak for not more than 2 minutes (10–12 sentences). Remember to say: what your typical school day is like; what your favourite subject is, and why what you like most about your school; what your attitude to your school life is. You have to talk continuously. Recording

Слайд 40

Task 3. You are going to give a talk about pets. You will have to start in 1.5 minutes and speak for not more than 2 minutes (10–12 sentences). Remember to say: why people keep pets; what pets are most popular in big cities; whether having pets is a big responsibility, and why; what your attitude to keeping pets is You have to talk continuously. Preparation

Слайд 41

Task 3. You are going to give a talk about pets. You will have to start in 1.5 minutes and speak for not more than 2 minutes (10–12 sentences). Remember to say: why people keep pets; what pets are most popular in big cities; whether having pets is a big responsibility, and why; what your attitude to keeping pets is You have to talk continuously. Recording

Слайд 42

Task 3. You are going to give a talk about travelling. You will have to start in 1.5 minutes and speak for not more than 2 minutes (10–12 sentences). Remember to say: why most people like travelling; which season is the best for travelling in your opinion; what means of transport is the best for travelling, and why; what your attitude to travelling is. You have to talk continuously. Preparation

Слайд 43

Task 3. You are going to give a talk about travelling. You will have to start in 1.5 minutes and speak for not more than 2 minutes (10–12 sentences). Remember to say: why most people like travelling; which season is the best for travelling in your opinion; what means of transport is the best for travelling, and why; what your attitude to travelling is. You have to talk continuously. Recording

Слайд 44

Task 3. You are going to give a talk about your school. You will have to start in 1.5 minutes and speak for not more than 2 minutes (10–12 sentences). Remember to say: what you like most about your school; what weekday you find the most difficult, and why; what you would like to change in your school life; what your attitude to your school life is. You have to talk continuously. Preparation

Слайд 45

Task 3. You are going to give a talk about your school. You will have to start in 1.5 minutes and speak for not more than 2 minutes (10–12 sentences). Remember to say: what you like most about your school; what weekday you find the most difficult, and why; what you would like to change in your school life; what your attitude to your school life is. You have to talk continuously. Recording

Слайд 46

Task 3. You are going to give a talk about learning foreign languages. You will have to start in 1.5 minutes and speak for not more than 2 minutes (10–12 sentences). Remember to say: why lots of people learn foreign languages nowadays; why you have chosen to do the English exam this year; what you did to prepare for your English exam; what your attitude to learning foreign languages is. You have to talk continuously. Preparation

Слайд 47

Task 3. You are going to give a talk about learning foreign languages. You will have to start in 1.5 minutes and speak for not more than 2 minutes (10–12 sentences). Remember to say: why lots of people learn foreign languages nowadays; why you have chosen to do the English exam this year; what you did to prepare for your English exam; what your attitude to learning foreign languages is. You have to talk continuously. Recording

Слайд 48

Task 3. You are going to give a talk about keeping fit. You will have to start in 1.5 minutes and speak for not more than 2 minutes (10–12 sentences). Remember to say: why a healthy lifestyle is popular nowadays; what you do to keep fit; what sports activities are popular with teenagers in your region; what your attitude to a healthy lifestyle is. You have to talk continuously. Preparation

Слайд 49

Task 3. You are going to give a talk about keeping fit. You will have to start in 1.5 minutes and speak for not more than 2 minutes (10–12 sentences). Remember to say: why a healthy lifestyle is popular nowadays; what you do to keep fit; what sports activities are popular with teenagers in your region; what your attitude to a healthy lifestyle is. You have to talk continuously. Recording

Слайд 50

Task 3. You are going to give a talk about the Internet. You will have to start in 1.5 minutes and speak for not more than 2 minutes (10–12 sentences). Remember to say: why today’s teenagers use the Internet so much; how the Internet makes long-distance communication easier; what dangers teenagers can face when they use the Internet; what your attitude to the Internet is. You have to talk continuously. Preparation

Слайд 51

Task 3. You are going to give a talk about the Internet. You will have to start in 1.5 minutes and speak for not more than 2 minutes (10–12 sentences). Remember to say: why today’s teenagers use the Internet so much; how the Internet makes long-distance communication easier; what dangers teenagers can face when they use the Internet; what your attitude to the Internet is. You have to talk continuously. Recording

Слайд 52

Task 3. You are going to give a talk about the place where you live. You will have to start in 1.5 minutes and speak for not more than 2 minutes (10–12 sentences). Remember to say: what your city, town or village is famous for; what your favourite place in your city, town or village is, and why you like it; whether you are going to stay in your city, town or village after leaving school or move to another place, and why; what your attitude to your city, town or village is. You have to talk continuously. Preparation

Слайд 53

Task 3. You are going to give a talk about the place where you live. You will have to start in 1.5 minutes and speak for not more than 2 minutes (10–12 sentences). Remember to say: what your city, town or village is famous for; what your favourite place in your city, town or village is, and why you like it; whether you are going to stay in your city, town or village after leaving school or move to another place, and why; what your attitude to your city, town or village is. You have to talk continuously. Recording

Слайд 54

27 Task 3. You are going to give a talk about your school. You will have to start in 1.5 minutes and speak for not more than 2 minutes (10–12 sentences). Remember to say: what your typical school day is like; what your favourite subject is, and why; what you like most about your school; what your attitude to your school life is. You have to talk continuously. Preparation

Слайд 55

27 Task 3. You are going to give a talk about your school. You will have to start in 1.5 minutes and speak for not more than 2 minutes (10–12 sentences). Remember to say: what your typical school day is like; what your favourite subject is, and why; what you like most about your school; what your attitude to your school life is. You have to talk continuously. Recording

Слайд 56

28 Task 3. You are going to give a talk about pets. You will have to start in 1.5 minutes and speak for not more than 2 minutes (10–12 sentences). Remember to say: why people keep pets; what pets are most popular in big cities; whether having pets is a big responsibility, and why; what your attitude to keeping pets is. You have to talk continuously. Preparation

Слайд 57

2 8 Task 3. You are going to give a talk about pets. You will have to start in 1.5 minutes and speak for not more than 2 minutes (10–12 sentences). Remember to say: why people keep pets; what pets are most popular in big cities; whether having pets is a big responsibility, and why; what your attitude to keeping pets is. You have to talk continuously. Recording

Слайд 58

29 Task 3. You are going to give a talk about travelling. You will have to start in 1.5 minutes and speak for not more than 2 minutes (10–12 sentences). Remember to say: why most people like travelling; which season is the best for travelling in your opinion; what means of transport is the best for travelling, and why; what your attitude to travelling is. You have to talk continuously. Preparation

Слайд 59

29 Task 3. You are going to give a talk about travelling. You will have to start in 1.5 minutes and speak for not more than 2 minutes (10–12 sentences). Remember to say: why most people like travelling; which season is the best for travelling in your opinion; what means of transport is the best for travelling, and why; what your attitude to travelling is. You have to talk continuously. Recording

Слайд 60

30 Task 3. You are going to give a talk about your school. You will have to start in 1.5 minutes and speak for not more than 2 minutes (10–12 sentences). Remember to say: what you like most about your school; what weekday you find the most difficult, and why; what you would like to change in your school life; what your attitude to your school life is. You have to talk continuously. Preparation

Слайд 61

30 Task 3. You are going to give a talk about your school. You will have to start in 1.5 minutes and speak for not more than 2 minutes (10–12 sentences). Remember to say: what you like most about your school; what weekday you find the most difficult, and why; what you would like to change in your school life; what your attitude to your school life is. You have to talk continuously. Recording

Слайд 62

Task 3. You are going to give a talk about learning foreign languages. You will have to start in 1.5 minutes and speak for not more than 2 minutes (10–12 sentences). Remember to say: why lots of people learn foreign languages nowadays; why you have chosen to do the English exam this year; what you did to prepare for your English exam; what your attitude to learning foreign languages is. You have to talk continuously. Preparation 31

Слайд 63

31 Task 3. You are going to give a talk about learning foreign languages. You will have to start in 1.5 minutes and speak for not more than 2 minutes (10–12 sentences). Remember to say: why lots of people learn foreign languages nowadays; why you have chosen to do the English exam this year; what you did to prepare for your English exam; what your attitude to learning foreign languages is. You have to talk continuously. Recording

Слайд 64

32 Task 3. You are going to give a talk about keeping fit. You will have to start in 1.5 minutes and speak for not more than 2 minutes (10–12 sentences). Remember to say: why a healthy lifestyle is popular nowadays; what you do to keep fit; what sports activities are popular with teenagers in your region; what your attitude to a healthy lifestyle is. You have to talk continuously. Preparation

Слайд 65

32 Task 3. You are going to give a talk about keeping fit. You will have to start in 1.5 minutes and speak for not more than 2 minutes (10–12 sentences). Remember to say: why a healthy lifestyle is popular nowadays; what you do to keep fit; what sports activities are popular with teenagers in your region; what your attitude to a healthy lifestyle is. You have to talk continuously. Recording

Слайд 66

33 Task 3. You are going to give a talk about the Internet. You will have to start in 1.5 minutes and speak for not more than 2 minutes (10–12 sentences). Remember to say: why today’s teenagers use the Internet so much; how the Internet makes long-distance communication easier; what dangers teenagers can face when they use the Internet; what your attitude to the Internet is. You have to talk continuously. Preparation

Слайд 67

33 Task 3. You are going to give a talk about the Internet. You will have to start in 1.5 minutes and speak for not more than 2 minutes (10–12 sentences). Remember to say: why today’s teenagers use the Internet so much; how the Internet makes long-distance communication easier; what dangers teenagers can face when they use the Internet; what your attitude to the Internet is. You have to talk continuously. Recording

Слайд 68

34 Task 3. You are going to give a talk about the place where you live. You will have to start in 1.5 minutes and speak for not more than 2 minutes (10–12 sentences). Remember to say: what your city, town or village is famous for; what your favourite place in your city, town or village is, and why you like it; whether you are going to stay in your city, town or village after leaving school or move to another place, and why; what your attitude to your city, town or village is. You have to talk continuously. Preparation

Слайд 69

34 Task 3. You are going to give a talk about the place where you live. You will have to start in 1.5 minutes and speak for not more than 2 minutes (10–12 sentences). Remember to say: what your city, town or village is famous for; what your favourite place in your city, town or village is, and why you like it; whether you are going to stay in your city, town or village after leaving school or move to another place, and why; what your attitude to your city, town or village is. You have to talk continuously. Recording

Слайд 70

https://vk.com/egexpert



Предварительный просмотр:

Ecology

Task 1. You are going to read the text aloud. You have 1.5 minutes to read the text silently, and then be ready to read it aloud. Remember that you will not have more than 2 minutes for reading aloud.

 

Recycling is a technology that helps protect the environment and cut down on usage of raw materials. The steel, paper and glass industries recycle a lot. The largest recycler is the steel industry. It recovers more than 70 per cent of its original materials. For example, since 1988 they have produced the majority of new metal cans from old ones. Metal parts for cars and planes are other examples of recycling steel. Fragments of waste glass are widely used in construction. For producing writing paper and pens, used packing boxes are an ideal material. They are cheap and easy to recycle. Nowadays more and more recycling centres are appearing in our towns and cities.

Task 1. You are going to read the text aloud. You have 1.5 minutes to read the text silently, and then be ready to read it aloud. Remember that you will not have more than 2 minutes for reading aloud.

 Nowadays solar energy is widely used as an alternative form of power. Solar panels transform the energy from the sun into electricity. The first plane that does not need fuel was constructed in France in 2015. It uses only the sun's energy. The panels are placed on the huge wings of the plane. It doesn’t fly very fast. Solar energy can make the plane move at only 140 miles an hour. However, the plane is able to travel round the world. It is safe and can successfully cross areas of bad weather. In the future, engineers hope to construct a model that people can fly in. Our dream of environmentally friendly transport may come true very soon. Would you like to take a flight on the solar plane?

Task 1. You are going to read the text aloud. You have 01.5 minutes to read the text silently, and then be ready to read it aloud. Remember that you will not have more than 2 minutes for reading aloud.

 Global warming is a result of human activity. In 1985 there was a big conference in Austria where 89 climate researchers from different countries took part. After long discussions the scientists declared that the climate of the planet had changed. Indeed, it is warmer now than it was years ago. It can be clearly seen in the north. In the Arctic some places are ice-free now and polar bears have lost the place where they lived. The sea level has risen as well. The situation is getting more and more serious. Global warming is a great problem which needs to be solved in the near future.

Task 1. You are going to read the text aloud. You have 01.5 minutes to read the text silently, and then be ready to read it aloud. Remember that you will not have more than 2 minutes for reading aloud.

 

We don’t realize how lucky we are to have clean water whenever we want. A lot of people around the world do not have such an opportunity. Did you know that more than 750 million people on the Earth do not have clean water to drink? Meanwhile, a lot of water is wasted and polluted. In 1993 the United Nations decided to celebrate the World Day for Water. Nowadays a lot of special events are organized in different countries. Their aim is to make people understand how important clean water is for their health, environment and agriculture. Water is essential for our life. We should think about saving it, as the fresh water supplies are limited. We need to keep it clean for ourselves and for future generations.

Task 1. You are going to read the text aloud. You have 01.5 minutes to read the text silently, and then be ready to read it aloud. Remember that you will not have more than 2 minutes for reading aloud.

 Earth Hour is an event that encourages people around the world to switch off electricity at the same time for one hour. Of course, switching off the lights for an hour makes a small difference to the amount of energy we use. But the aim of the event is not to save energy but to get people to think about the planet they live on. Earth Hour started in Australia in 2007. More and more countries join the ecological movement every year. Now, millions of people from 178 countries switch off their lights for 60 minutes at the end of March. People in Russia participate in this event too –– large office buildings and small private houses go dark for Earth Hour.

Task 1. You are going to read the text aloud. You have 01.5 minutes to read the text silently, and then be ready to read it aloud. Remember that you will not have more than 2 minutes for reading aloud.

 Human activity creates waste. The problem isn’t new and space is no exception to the rule. Dead satellites, rocket stages and other things are littering our planet’s orbit. They are getting dangerously close to space stations and there is a great possibility of an accident. Space stations could be completely destroyed by the rubbish. In 1983, a tiny piece of paint from a satellite made a big hole in the side window of a spaceship. There is a great need to “clean up” space and to take away 3,000 tons of space rubbish. To solve the problem, scientists and engineers are designing special spaceships that will collect and recycle space rubbish.

Task 1. You are going to read the text aloud. You have 01.5 minutes to read the text silently, and then be ready to read it aloud. Remember that you will not have more than 2 minutes for reading aloud.

 

The rainforest can be described as a thick and very tall jungle. The term rain comes from the great quantity of water that these forests get throughout the year. The rainforests are the world's greatest natural resources. They are called the lungs of our planet. Half of all the kinds of plants and animals that exist on the planet live in the rainforests. Unfortunately, the area with rainforests is being reduced due to global warming. 100 years ago, the rainforests covered 14 per cent of the earth’s surface. Now, it is only 6 per cent. Scientists say that if the process continues at this rate, the rainforests will have disappeared in 40 years.

Task 1. You are going to read the text aloud. You have 01.5 minutes to read the text silently, and then be ready to read it aloud. Remember that you will not have more than 2 minutes for reading aloud.

 A zoo, also known as a zoological park or a zoological garden, gives people the opportunity to watch animals from different continents. There are hundreds of zoos all over the world. However, most of them are located in major cities. The first zoo in Russia was opened in 1864, in Moscow. It was a small zoological garden. On the opening day, only 287 animals were on display. Nowadays, the Moscow Zoo is one of the largest in Europe. The unique collection of the Zoo includes over 8000 kinds of animals and birds. Scientists study the animals’ behavior and reproduction. They also breed rare animals there to stop them dying out.

Space

Task 1. You are going to read the text aloud. You have 1.5 minutes to read the text silently, and then be ready to read it aloud. Remember that you will not have more than 2 minutes for reading aloud.

 Do you know why it is hot in the summer and cold in the winter? Most people believe that it is because the Earth is closer to the Sun in the summer and farther from it in the winter. But it is not like this at all! In fact, the distance between the Earth and the Sun is largest in July and smallest in January! Scientists suggest that the reason for different summer and winter temperatures is the angle of the Sun’s rays. In the summer, the rays reach the Earth almost at a right angle. Due to this, the rays are not spread out and a smaller area of the planet's surface gets the energy. In the winter, the angle changes and the Sun’s rays are spread out over a larger area. Besides this, the long nights and short days do not give enough time for the land to warm up.

Task 1. You are going to read the text aloud. You have 1.5 minutes to read the text silently, and then be ready to read it aloud. Remember that you will not have more than 2 minutes for reading aloud.

 Without the energy from the Sun, the Earth would have no life at all. Nowadays everyone knows that the Sun is a star. Like all stars, the Sun is a great burning ball of gases. To us, it seems large and red. The other stars look white or light blue because they are much farther from us than the Sun. In the past, many people believed that the Earth was the centre of the universe. They thought that the Sun and the stars went round it. Only in 1543, a great Polish scientist published a book in which he tried to prove a different point of view. According to him, the Earth and the other planets moved round the Sun. Those people who supported the scientist were often imprisoned and even killed.

Task 1. You are going to read the text aloud. You have 1.5 minutes to read the text silently, and then be ready to read it aloud. Remember that you will not have more than 2 minutes for reading aloud.

 Venus is one of the hottest planets in the Solar system. The temperature on its surface is 482 degrees. This unfriendly place looks much the same as the Earth 4 billion years ago. Both planets were formed from the same gas cloud, but Venus lost most of its water and atmosphere. This happened because the planet turns very slowly. As a result, now it has no magnetic field to protect it from the Sun's winds. The planet has turned into a lifeless desert. Most space modules that were sent to Venus, could not work on the planet more than two hours. The acidic clouds and high pressure quickly made them absolutely useless.

Task 1. You are going to read the text aloud. You have 1.5 minutes to read the text silently, and then be ready to read it aloud. Remember that you will not have more than 2 minutes for reading aloud.

 Most people enjoy watching the night sky. And the brightest object in the sky is the Moon. Every night the Moon rises in the East and sets in the West. It takes about 27 days to go around the Earth, and it’s an interesting fact that we always see the same side of the Moon. The Moon is the closest object to the Earth. It would take us only 13 hours to get to the Moon by rocket. If we were able to travel at the speed of light, it would take a bit less than two seconds.

There is a very thin atmosphere on the Moon. Life is not possible there. In spite of this, the Moon can be used as a base for space exploration and even for space tourism. Some space companies say that they will be able to take tourists to the Moon as soon as in 2020.

Task 1. You are going to read the text aloud. You have 1.5 minutes to read the text silently, and then be ready to read it aloud. Remember that you will not have more than 2 minutes for reading aloud.

 The ninth planet of the solar system was discovered not long ago. It happened in 1930. Scientists had been hunting for the planet for a long time. They had calculated its probable position, but there was no proof that the planet really existed. It was too far away for the telescopes of that time to find it. It’s worth mentioning that the first photos of the planet were taken by a very young researcher. He was only twenty-four and had no formal education in astronomy. However he was deeply involved in the search for the ninth planet. The planet at the edge of the solar system was called Pluto, after the Roman god. The name for the planet was suggested by an 11-year-old British girl.

Technological progress

Task 1. You are going to read the text aloud. You have 1.5 minutes to read the text silently, and then be ready to read it aloud. Remember that you will not have more than 2 minutes for reading aloud.

 A robot is a machine that does work for people. The word ‘robot’ appeared in the 1920s and it was first used in a science fiction story. In real life, the robot was invented later, in 1954. There are different kinds of robots which are used in different industries. Most of them are operated with the help of a computer program. Instead of humans, robots do hard and boring work in unpleasant or dangerous environments. They usually work faster and more accurately than people. They never get tired and do not make mistakes. Nowadays robots are often used for domestic needs –– to clean houses or to look after sick and elderly people. Scientists say that soon robots will become a normal part of our life, like mobiles and computers today.

Task 1. You are going to read the text aloud. You have 1.5 minutes to read the text silently, and then be ready to read it aloud. Remember that you will not have more than 2 minutes for reading aloud.

 

Nowadays people can’t imagine their kitchen without a refrigerator. The first refrigerators appeared at the end of the 18th century. They looked different: they were just iceboxes. The refrigerators of the past were just wooden boxes, sometimes lined inside with metal. People put ice into the box and then placed the food inside to keep it cool. The refrigerator, similar to the one that we have now, was produced by General Electric in 1911, in France. It was very expensive. In those days, with the same money you could buy two cars. Today, in developed countries almost every family has enough money to buy a refrigerator for their kitchen. It’s hard to imagine how difficult and uncomfortable our lives would be without a refrigerator.

Task 1. You are going to read the text aloud. You have 1.5 minutes to read the text silently, and then be ready to read it aloud. Remember that you will not have more than 2 minutes for reading aloud.

 Since 2007 high-speed trains have become a common type of transport in Europe. They make it possible to cut travel times by hours. The trains travel at 350 kilometres an hour, and this is not the limit! The only problem for the engineers is the loud noise the trains produce. People living in the areas the trains travel through suffer from the noise greatly. For the passengers, the situation is different. For their comfort, all the carriages are made sound-proof from the inside. Travelling on high-speed trains is comfortable and pleasant. In the future, a high-speed railroad network could cover most of Europe. For example, a trip from Paris to Rome could take only 3 hours!

Task 1. You are going to read the text aloud. You have 1.5 minutes to read the text silently, and then be ready to read it aloud. Remember that you will not have more than 2 minutes for reading aloud.

 In 1642 a French student invented the first mechanical calculator. He was only 18 and the counting machine was a present for his father. The father worked as a tax collector and the young man wanted to make his job easier.

The talented inventor made several machines, but nobody was interested. The young man was ahead of his time. Many years passed before people realized how useful these machines could be and started mass producing them.

Today most mechanical calculators have been replaced with electronic models. These small devices can perform a lot of mathematical operations. You just need to enter the numbers by pressing the keys and you’ll see the final result on the screen in no time.

Task 1. You are going to read the text aloud. You have 1.5 minutes to read the text silently, and then be ready to read it aloud. Remember that you will not have more than 2 minutes for reading aloud.

 

The first vacuum cleaner was designed in 1869. Since then, vacuum cleaners have changed a lot. Nowadays the robot vacuum cleaner is getting more and more popular. This device cleans a house automatically. Following a programme, the robot drives slowly around the house and makes it cleaner. It operates with brushes and a tube. The robot is so flexible that it can get under a table or sofa or reach dust in corners. Some advanced models can also wash floors of all kinds. These functions are provided by a complicated electronic system and sensors inside. The vacuum cleaner looks like a plate and can be kept in the corner. It is friendly to children and animals and works on electricity.

Task 1. You are going to read the text aloud. You have 01.5 minutes to read the text silently, and then be ready to read it aloud. Remember that you will not have more than 2 minutes for reading aloud.

 The mobile phone has become an important part of our everyday life. We can’t imagine our lives without it now. Thanks to the mobile phone we can keep in contact with our relatives and friends at any time. We use them to text and to send emails, to share photos and videos. We can use the Internet and listen to music and do lots of other things. However, according to surveys, mobile phone users mostly spend their time on games and social networking. Around 80 percent of the world’s population has a mobile phone. The mobile phone industry is the fastest growing industry in the world. From 1983 to 2016, worldwide mobile phone subscriptions grew to over 7 billion.

Our Planet

Task 1. You are going to read the text aloud. You have 1.5 minutes to read the text silently, and then be ready to read it aloud. Remember that you will not have more than 2 minutes for reading aloud.

 Snowdon is the highest mountain in Wales. It is located in a national park. It stands at 1085 metres above sea level, and it is often described as the busiest mountain in Great Britain. Snowdon is very popular with tourists. If you are strong and brave enough to get to the top, you can enjoy wonderful views from there. On a clear day, you can see as far as Ireland! During the summer months there is a café at the top. There, tourists can get a welcome cup of tea, or soup if the weather is cold. If you feel too tired to walk back, you can always take the train down the mountain. The mountain railway was built in 1896. It is safe, and there have never been any accidents on this route.

Task 1. You are going to read the text aloud. You have 1.5 minutes to read the text silently, and then be ready to read it aloud. Remember that you will not have more than 2 minutes for reading aloud.

 Studies of the atmosphere first received technical support in 1912. Equipment to measure temperature and pressure was invented in Germany. But the question was how to raise it high into the air. In 1921, Russian engineers suggested using planes. The special equipment was put on planes that made regular flights. Thanks to this, scientists got a lot of new information about the structure of the atmosphere. Nowadays scientists use modern flying laboratories to study the structure of clouds at different levels. Unlike the first planes, these laboratories can work in any weather and are able to predict climate changes.

Task 1. You are going to read the text aloud. You have 1.5 minutes to read the text silently, and then be ready to read it aloud. Remember that you will not have more than 2 minutes for reading aloud.

 The word ‘tsunami’ can be translated from Japanese as ‘big wave’. It describes a natural process that can be dangerous for people and their homes. Most of the huge waves appear after earthquakes. Most waves are born in the Indian and the Pacific Oceans where volcanoes are active. The mass of water rises from the bottom of the ocean and moves to the shore. It moves at a speed of a plane and can be up to 40 meters high. The wave is very powerful and dangerous. In 2004, a tsunami happened in the Indian Ocean. It was one of most terrible natural disasters in history. It hit 14 countries bordering the Indian Ocean. Thousands of people were killed or went missing.

Task 1. You are going to read the text aloud. You have 1.5 minutes to read the text silently, and then be ready to read it aloud. Remember that you will not have more than 2 minutes for reading aloud.

 People have always wanted to discover new things and new worlds. We have built spaceships to go to other planets and are planning to visit other galaxies one day. Meanwhile, we know very little about our own planet. The oceans, which cover 70 percent of the planet, are an unknown world to us. The Pacific Ocean is the largest one. The lowest point on the Earth is also there. It is called Challenger Deep and it was discovered in 1875. The Pacific Ocean stretches from the Arctic Ocean to the coast of Antarctica, but most of its water is in the warm tropics, which makes it warm. A lot of fish and animals live in the Pacific Ocean. The place is still waiting for researchers to discover its secrets for us.

Task 1. You are going to read the text aloud. You have 1.5 minutes to read the text silently, and then be ready to read it aloud. Remember that you will not have more than 2 minutes for reading aloud.

 

The Arctic Ocean is the smallest ocean of the world. It covers the northern polar region of the Earth. The Arctic region is a very cold place. The lowest recorded temperature there is minus 68 degrees. In spite of the cold, a lot of animals live on the Arctic ice. The polar bear is one of them. Now, due to global warming, the polar bear is in a serious trouble. The thing is that polar bears hunt seals and they need ice to do this. The bears wait for seals on the ice, hunt them in the water and go back to the shore. If the ice is too thin, it breaks under the bears’ heavy bodies and they may drown. The situation is getting worse and some researchers say the ocean may become ice-free as soon as 2050.

Task 1. You are going to read the text aloud. You have 01.5 minutes to read the text silently, and then be ready to read it aloud. Remember that you will not have more than 2 minutes for reading aloud.

 Our planet is the only planet in the Solar System where rainbows are possible. They are not visible in big cities, due to pollution and smog. But if you are travelling after rain has fallen in the countryside or walking on the beach, you have a good chance to see a rainbow. The rainbow is a multi-coloured arc which appears in the sky. When you see a rainbow, the sun is always behind you and it is raining in front of you. It is impossible to get to the end of the rainbow and to touch it. When you move, the rainbow moves too. We have always learned that rainbows have 7 colours. However, until the 17th century, people used to think there were only 5 colours in the rainbow.

Task 1. You are going to read the text aloud. You have 01.5 minutes to read the text silently, and then be ready to read it aloud. Remember that you will not have more than 2 minutes for reading aloud.

 

The Caspian Sea is one of the world’s natural wonders. It shares the characteristics of both a sea and a lake. In the past it was called a sea because of its great size and salty water. However, the Caspian Sea is listed as the largest lake in the world. There are about 130 rivers which bring their water to the Caspian Sea. But it is not a freshwater lake –– its water is very salty. The Caspian Sea is famous for its wildlife. More than 850 kinds of animals and 500 different plants can be found in this unique sea. The Caspian Sea region is also one of the oldest oil-producing areas in the world. It is also rich in other natural resources.

Task 1. You are going to read the text aloud. You have 1.5 minutes to read the text silently, and then be ready to read it aloud. Remember that you will not have more than 2 minutes for reading aloud.

 

Wombats are exotic animals that only live in Australia. They have become an unofficial symbol of the country. In 1974 a wombat appeared on an Australian stamp for the first time. Since then it has been regularly used on different Australian stamps and coins. Wombats look like little bears and prefer grass to meat. They also love eating berries, plant roots and mushrooms. Like camels, wombats drink little water because there is enough water in fresh grass. Wombats spend most of their life in holes under the ground. With their strong arms and long nails they are able to dig very long and complex tunnels. Fortunately, wombats have few enemies in the animal world. Most wombats live around 15 years.

Sport

Task 1. You are going to read the text aloud. You have 1.5 minutes to read the text silently, and then be ready to read it aloud. Remember that you will not have more than 2 minutes for reading aloud.

 

One day a young man who lived near the lake in a small American town asked himself, “Why can’t people ski on water if they can ski on snow?” With his brother’s help he constructed several models of skis to test his ideas. He experimented on the local lake for a few days and created skis suitable for the water. This happened in 1922. The young man, who was only 18 at that time, didn’t patent his invention. However, he performed a lot of shows across the country that made him and the sport of water skiing popular. Later, he was recognized as the creator of a new sport –– water skiing.

Water skiing equipment has been greatly improved since that time, and the number of people who enjoy this sport is growing in many countries.

Task 1. You are going to read the text aloud. You have 01.5 minutes to read the text silently, and then be ready to read it aloud. Remember that you will not have more than 2 minutes for reading aloud.

 

Netball is played by over 20 million people in more than 70 countries of the world. It was invented in 1892 as women’s basketball. However, netball is different from basketball in many ways. For example, the ball and the basket are smaller and the court is bigger. The players cannot run with the ball. They are not allowed to touch the players who they are playing with. Netball has seven players in each team (not five as in basketball). During the 20th century, the game became very popular in English-speaking countries. Traditionally, only girls and women have played netball. Nowadays, netball is the number one women’s sport in the world but boys and men are starting to play it too.

Task 1. You are going to read the text aloud. You have 01.5 minutes to read the text silently, and then be ready to read it aloud. Remember that you will not have more than 2 minutes for reading aloud.

 Table tennis is the third most played sport in the world after football and cricket. It is considered to be an indoor version of Lawn Tennis. However, they have a lot of differences. Table tennis has a different scoring system, a smaller ball, smaller rackets, and a smaller playing area. Modern table tennis was invented at the end of the 19th century in England. Very soon the new sport became popular all over the world. In 1901, competitions with over 300 participants were already being held. According to numerous studies, table tennis has great effects on human health. It keeps the brain sharp and improves the player’s coordination. Since 1988, table tennis has been an Olympic sport.

Studying

Task 1. You are going to read the text aloud. You have 01.5 minutes to read the text silently, and then be ready to read it aloud. Remember that you will not have more than 2 minutes for reading aloud.

 English is the third most-spoken language in the world after Chinese and Spanish. Over 840 million people speak English as a first or second language. It is an official language of 67 countries. 80 percent of information stored on all computers in the world is in English. The vocabulary of the English language is the biggest in the world. A new word is added into English almost every two hours. The English language is said to be one of the happiest languages in the world. The word happy is used more often than the word sad! That's a good reason to learn this language, isn’t it?

Task 1. You are going to read the text aloud. You have 01.5 minutes to read the text silently, and then be ready to read it aloud. Remember that you will not have more than 2 minutes for reading aloud.

 The human brain is the most complex and remarkable organ of the body. It controls everything we do –– thinking, learning, and even heart rate. Though the brain makes up only 2 percent of the body's weight, it uses around 20 percent of its energy. It works 24 hours a day. Even when a person is sleeping, the brain is working hard to keep other organs functioning. Most people think that our brain is more active during the daytime because it needs to deal with a great amount of information. In fact, that’s not true. The brain is far more active when a person is sleeping! However, scientists don’t yet have any explanation for this fact.

Task 1. You are going to read the text aloud. You have 01.5 minutes to read the text silently, and then be ready to read it aloud. Remember that you will not have more than 2 minutes for reading aloud.

 Great Britain is home to the world’s oldest school. The King’s School was established in 1567 and it is still open. It has a long history. Many famous people have been educated at this old school. Nowadays The King’s School is an independent secondary school for boys and girls from 13 to 18. More than 800 pupils study at the school. Though The King’s School is located in old buildings, it has up-to-date technical equipment and provides a full modern education. The King’s School is also famous for its festival of music, drama and arts. It is held during the last week of the summer term and attracts a lot of people from different parts of the country.

Task 1. You are going to read the text aloud. You have 01.5 minutes to read the text silently, and then be ready to read it aloud. Remember that you will not have more than 2 minutes for reading aloud.

 Oxford University is the oldest university in the English-speaking world. It is also the second oldest university in the world. The University of Oxford is believed to have started in 1133. It is located in Oxford City on the Thames River, not far from London. Oxford University is one of the world’s most impressive centres of education. There are students from different countries there. In fact, there is no building in Oxford called Oxford University. The University includes 38 colleges and related buildings. Over 22,000 students study English language and literature, geography, history, law, modern languages and other subjects there.

Health

Task 1. You are going to read the text aloud. You have 1.5 minutes to read the text silently, and then be ready to read it aloud. Remember that you will not have more than 2 minutes for reading aloud.

 

We are always told that we should clean our teeth regularly to keep them healthy and to have fresh breath. It is believed that people started using a kind of paste to clean their teeth around 5000 years ago. However, the ingredients of these tooth powders were very different from ours. For example, the people of ancient Egypt used salt, mint, dried flowers and pepper to create tooth powder. Later, in the 18th century, in some countries in Europe, people brushed their teeth with burnt bread. The first toothpaste appeared in 1890, in Great Britain. At first it was sold in jars. Then special tubes were designed to make the toothpaste more comfortable to use.

Food

Task 1. You are going to read the text aloud. You have 1.5 minutes to read the text silently, and then be ready to read it aloud. Remember that you will not have more than 2 minutes for reading aloud.

 

The carrot is a root vegetable which consists mostly of water and sugar. The carrots that we eat today originated from the wild ones which grew in Europe and Asia. The vegetable tasted sweet and the people began to use it as food.

There are two main types of carrots. There are eastern carrots that have dark purple roots and western carrots with orange roots. In fact, orange carrots first appeared in Europe in 1721. Dutch farmers selected orange carrots because orange is the colour of the Dutch Royal Family. Now orange carrots are widely grown all over the world. In modern households carrots are cooked and eaten in different ways. The vegetable is boiled, fried, baked and eaten raw. People also make juice from it.

Task 1. You are going to read the text aloud. You have 01.5 minutes to read the text silently, and then be ready to read it aloud. Remember that you will not have more than 2 minutes for reading aloud.

 

Though most people believe that the tomato is a vegetable, this plant is actually a fruit. Tomato is a fruit because it has seeds inside like apples, oranges, lemons, and all other types of fruit. The tomato comes from Central and South America. When tomatoes were brought to England in the 17th century, people thought that they were poisonous and refused to eat them. Only in 1820, Robert Johnson proved that tomatoes were absolutely safe and would not affect people’s health. He ate a basket of tomatoes in public. Nowadays, the tomato is widely used in all types of food. There are many different kinds of tomatoes –– over 7,500! They differ in size, shape, colour and taste.

Culture

Task 1. You are going to read the text aloud. You have 01.5 minutes to read the text silently, and then be ready to read it aloud. Remember that you will not have more than 2 minutes for reading aloud.

 

It's hard to find a more popular Russian symbol than the traditional Russian doll, or Matryoshka. It is well-known all over the world. The first set of dolls appeared in Moscow in 1890. It consisted of eight dolls. All of them were children: seven girls, one boy, and a baby. In those times, Matryoshkas were not meant as toys for children as their price was very high. Mostly, adults bought them as presents or home decorations. There was a belief that if you put a note with a wish into Matryoshka, it would come true. At the beginning of the 20th century, Russia started to export their national dolls abroad.

Transport

Task 1. You are going to read the text aloud. You have 01.5 minutes to read the text silently, and then be ready to read it aloud. Remember that you will not have more than 2 minutes for reading aloud.

 

The construction of the longest and the deepest railroad tunnel was finished in Europe. The tunnel runs through the Alps and links the north and south of Europe. The length of the tunnel is 57 kilometres. It runs straight from beginning to end. It will allow passenger trains to travel more quickly through the mountains. Modern technologies ensure safety and security when the trains go through the tunnel. The great project started in 2009 and it took 17 years to complete. It is considered to be one of the most amazing engineering constructions in Europe.

Task 1. You are going to read the text aloud. You have 01.5 minutes to read the text silently, and then be ready to read it aloud. Remember that you will not have more than 2 minutes for reading aloud.

 

Bicycles or bikes are an important means of transportation in many parts of the world. The first bicycles turned up in Europe in the first half of the19th century but the word ‘bicycle’ only appeared later, in 1868. There are more bicycles in the world than cars. About 100 million bicycles are produced worldwide each year. A lot of people have realized that cycling is an easy way to get around and a great way to cut down on pollution. More and more cities have special places where people can borrow a bike and ride around the city. Over the past few years, a bicycle infrastructure has been created in Moscow. Cycling in the centre of Moscow in summer is one of the most pleasant and quickest ways of seeing the city.

Task 1. You are going to read the text aloud. You have 01.5 minutes to read the text silently, and then be ready to read it aloud. Remember that you will not have more than 2 minutes for reading aloud.

 

The Channel Tunnel is a rail tunnel beneath the English Channel which links Great Britain and France. It is considered to be one of the most amazing engineering constructions in Europe. The second longest undersea tunnel in the world was opened in 1994. It is said that 13,000 workers and engineers participated in the construction project. The Channel Tunnel contains three tunnels. The south tunnel is for people who are travelling from France to the UK, the north tunnel takes passengers from the UK to France. Nowadays it takes 35 minutes to go from Paris to London in the Channel Tunnel. The third tunnel is a service tunnel. It is not used by passengers.



Предварительный просмотр:

You have 30 minutes to do this task.

You have received a letter from your English-speaking pen friend, Emily.

… We are having a Maths test tomorrow. It makes me feel scared! I would rather have three language tests instead of one Maths test. I really find learning foreign languages interesting and not difficult at all. ...

…What subject is the most difficult for you? What do you do to improve your English? What language apart from English would you like to learn and why?...

Write her a letter and answer her 3 questions.

Write 100–120 words. Remember the rules of letter writing.

                                                                                     Moscow

                                                                                      Russia

                                                                                  20/01/2020

Dear Emily,

Thanks a lot for your letter. It was great to hear from you again. I hope you’re doing fine.

In your letter you ask me about school subjects. Just like you, I enjoy learning foreign languages. English is my favourite subject! To improve my English language skills, I do lots of exercises, watch YouTube videos, read blogs and listen to podcasts in English. I’d be happy to learn other foreign languages, too. For example, I’d love to learn Spanish because it’s a beautiful language and I’ve always dreamed of visiting Spain. It’d be great if I could have classes of Spanish instead of Maths. Maths seems to me the hardest school subject.

Write again when you have time.

Best wishes,

Zhenya

Адрес и дата в верхнем правом углу

Обращение,

Благодарность за полученное письмо.

Ссылка на предыдущие контакты.

Ответы на три вопроса:

2) What do you do to improve your English?

3) What language apart from English would you like to learn and why?

  1. What subject is the most difficult for you?

Надежда на последующие контакты.

Завершающая фраза,

Подпись